Docsity
Docsity

Prepare for your exams
Prepare for your exams

Study with the several resources on Docsity


Earn points to download
Earn points to download

Earn points by helping other students or get them with a premium plan


Guidelines and tips
Guidelines and tips

Charmberlain college of nursing NR508 Final Questions with Verified Answers Latest Updat, Exams of Nursing

Charmberlain college of nursing NR508 Final Questions with Verified Answers Latest Updated solutions

Typology: Exams

2022/2023

Available from 10/09/2022

hesigrader002
hesigrader002 🇺🇸

4.1

(38)

1.7K documents

1 / 92

Toggle sidebar

Related documents


Partial preview of the text

Download Charmberlain college of nursing NR508 Final Questions with Verified Answers Latest Updat and more Exams Nursing in PDF only on Docsity! Chapter 1 Charmberlain college of nursing NR508 Final Questions with Verified Answers Latest Updated solutions 1.Nurse practitioner prescriptive authority is regulated by: 1 The National Council of State Boards of Nursing . 2 The U.S. Drug Enforcement Administration . 3 The State Board of Nursing for each state . 4 The State Board of Pharmacy . 2.The benefits to the patient of having an Advanced Practice Registered Nurse (APRN) prescriber include: 1. Nurses know more about Pharmacology than other prescribers because they take it both in their basic nursing program and in their APRN program. 2. Nurses care for the patient from a holistic approach and include the patient in decision making regarding their care. 3. APRNs are less likely to prescribe narcotics and other controlled substances. 4. APRNs are able to prescribe independently in all states, whereas a physician’s assistant needs to have a physician supervising their practice. 3.Clinical judgment in prescribing includes: 1. Factoring in the cost to the patient of the medication prescribed 2. Always prescribing the newest medication available for the disease process 3. Handing out drug samples to poor patients 4. Prescribing all generic medications to cut costs 4.Criteria for choosing an effective drug for a disorder include: 1. Asking the patient what drug they think would work best for them 2. Consulting nationally recognized guidelines for disease management 3. Prescribing medications that are available as samples before writing a prescription 4. Following U.S. Drug Enforcement Administration guidelines for prescribing 5.Nurse practitioner practice may thrive under health-care reform because of: 1. The demonstrated ability of nurse practitioners to control costs and improve patient outcomes 2. The fact that nurse practitioners will be able to practice independently 3. The fact that nurse practitioners will have full reimbursement under health- care reform 4. The ability to shift accountability for Medicaid to the state level Chapter 2. Review of Basic Principles of Pharmacology 1.A patient’s nutritional intake and laboratory results reflect hypoalbuminemia. This is critical to prescribing because: 1. Distribution of drugs to target tissue may be affected. 2. The solubility of the drug will not match the site of absorption. 3. There will be less free drug available to generate an effect. 4. Drugs bound to albumin are readily excreted by the kidneys. 2. Drugs that have a significant first-pass effect: 1. Must be given by the enteral (oral) route only 2. Bypass the hepatic circulation 3. Are rapidly metabolized by the liver and may have little if any desired action 4. Are converted by the liver to more active and fat-soluble forms 3.The route of excretion of a volatile drug will likely be the: 1. Kidneys 2. Lungs 3. Bile and feces 4. Skin 4.Medroxyprogesterone (Depo Provera) is prescribed intramuscularly (IM) to create a storage reservoir of the drug. Storage reservoirs: 1. Assure that the drug will reach its intended target tissue 2. Are the reason for giving loading doses 3. Increase the length of time a drug is available and active 4. Are most common in collagen tissues 5.The NP chooses to give cephalexin every 8 hours based on knowledge of the drug’s: 1. Propensity to go to the target receptor 2. Biological half-life 3. Pharmacodynamics 4. Safety and side effects 6.Azithromycin dosing requires that the first day’s dosage be twice those of the other 4 days of the prescription. This is considered a loading dose. A loading dose: 1. Rapidly achieves drug levels in the therapeutic range 2. Requires four- to five-half-lives to attain 3. Is influenced by renal function 4. Is directly related to the drug circulating to the target tissues 7.The point in time on the drug concentration curve that indicates the first sign of a therapeutic effect is the: 1. Minimum adverse effect level 2. Peak of action 3. Onset of action 4. Therapeutic range 8.Phenytoin requires that a trough level be drawn. Peak and trough levels are done: 1. When the drug has a wide therapeutic range 2. When the drug will be administered for a short time only 3. When there is a high correlation between the dose and saturation of receptor sites 4. To determine if a drug is in the therapeutic range 9.A laboratory result indicates that the peak level for a drug is above the minimum toxic concentration. This means that the: 1. Concentration will produce therapeutic effects 2. Half-life 3. Phase II metabolism 4. Reduced bioavailability time 25.An agonist activates a receptor and stimulates a response. When given frequently over time, the body may: 1. Upregulate the total number of receptors 2. Block the receptor with a partial agonist 3. Alter the drug’s metabolism 4. Downregulate the numbers of that specific receptor 26.Drug antagonism is best defined as an effect of a drug that: 1. Leads to major physiological and psychological dependence 2. Is modified by the concurrent administration of another drug 3. Cannot be metabolized before another dose is administered 4. Leads to a decreased physiological response when combined with another drug 27. Instructions to a client regarding self-administration of oral enteric-coated tablets should include which of the following statements? 1. “Avoid any other oral medicines while taking this drug.” 2. “If swallowing this tablet is difficult, dissolve it in 3 ounces of orange juice.” 3. “The tablet may be crushed if you have any difficulty taking it.” 4. “To achieve best effect, take the tablet with at least 8 ounces of fluid.” 28.The major reason for not crushing a sustained-release capsule is that, if crushed, the coated beads of the drugs could possibly result in: 1. Disintegration 2. Toxicity 3. Malabsorption 4. Deterioration 29.Which of the following substances is the most likely to be absorbed in the intestines rather than in the stomach? 1. Sodium bicarbonate 2. Ascorbic acid 3. Salicylic acid 4. Glucose 30.Which of the following variables is a factor in drug absorption? 1. The smaller the surface area for absorption, the more rapidly the drug is absorbed. 2. A rich blood supply to the area of absorption leads to better absorption. 3. The less soluble the drug, the more easily it is absorbed. 4. Ionized drugs are easily absorbed across the cell membrane. 31.An advantage of prescribing a sublingual medication is that the medication is: 1. Absorbed rapidly 2. Excreted rapidly 3. Metabolized minimally 4. Distributed equally 32.Drugs that use CYP 3A4 isoenzymes for metabolism may: 1. Induce the metabolism of another drug 2. Inhibit the metabolism of another drug 3. Both 1 and 2 4. Neither 1 nor 2 33.Therapeutic drug levels are drawn when a drug reaches steady state. Drugs reach steady state: 1. After the second dose 2. After four to five half-lives 3. When the patient feels the full effect of the drug 4. One hour after IV administration 34.Upregulation or hypersensitization may lead to: 1. Increased response to a drug 2. Decreased response to a drug 3. An exaggerated response if the drug is withdrawn 4. Refractoriness or complete lack of response Chapter 3. Rational Drug Selection 1.An NP would prescribe the liquid form of ibuprofen for a 6-year-old child because: 1. Drugs given in liquid form are less irritating to the stomach. 2. A 6-year-old child may have problems swallowing a pill. 3. Liquid forms of medication eliminate the concern for first-pass effect. 4. Liquid ibuprofen does not have to be dosed as often as the tablet form. 2.In deciding which of multiple drugs used to use to treat a condition, the NP chooses Drug A because it: 1. Has serious side effects and it is not being used for a life-threatening condition 2. Will be taken twice daily and will be taken at home 3. Is expensive, but covered by health insurance 4. None of these are important in choosing a drug 3.A client asks the NP about the differences in drug effects between men and women. What is known about the differences between the pharmacokinetics of men and women? 1. Body temperature varies between men and women. 2. Muscle mass is greater in women. 3. Percentage of fat differs between genders. 4. Proven subjective factors exist between the genders. 4.The first step in the prescribing process according to the World Health Organization is: 1. Choosing the treatment 2. Educating the patient about the medication 3. Diagnosing the patient’s problem 4. Starting the treatment 5.Treatment goals in prescribing should: 1. Always be curative 2. Be patient-centered 3. Be convenient for the provider 4. Focus on the cost of therapy 6.The therapeutic goals when prescribing include(s): 1. Curative 2. Palliative 3. Preventive 4. All of the above 7.When determining drug treatment the NP prescriber should: 1. Always use evidence-based guidelines 2. Individualize the drug choice for the specific patient 3. Rely on his or her experience when prescribing for complex patients 4. Use the newest drug on the market for the condition being treated 8.Patient education regarding prescribed medication includes: 1. Instructions written at the high school reading level 2. Discussion of expected adverse drug reactions 3. How to store leftover medication such as antibiotics 4. Verbal instructions always in English 9.Passive monitoring of drug effectiveness includes: 1. Therapeutic drug levels 2. Adding or subtracting medications from the treatment regimen 3. Ongoing provider visits 4. Instructing the patient to report if the drug is not effective 10.Pharmacokinetic factors that affect prescribing include: 1. Therapeutic index 2. Minimum effective concentration 3. Bioavailability 4. Ease of titration 11.Pharmaceutical promotion may affect prescribing. To address the impact of pharmaceutical promotion, the following recommendations have been made by the Institute of Medicine: 1. Conflicts of interest and financial relationships should be disclosed by those providing education. 2. Providers should ban all pharmaceutical representatives from their office setting. 3. Drug samples should be used for patients who have the insurance to pay for them, to ensure the patient can afford the medication. 4. Providers should only accept low-value gifts, such as pens and pads of paper, from the pharmaceutical representative. 12.Under new U.S. Food and Drug Administration labeling, Pregnancy Categories will be: 1. Strengthened with a new coding such as C+ or C- to discern when a drug is more or less toxic to the fetus 2. Changed to incorporate a pregnancy risk summary and clinical considerations on the drug label 3. The blood-brain barrier being less permeable, requiring higher doses to achieve therapeutic effect 4. Age-related decrease in renal function 4.The type of adverse drug reaction that is idiosyncratic when a drug given in the usual therapeutic doses is type: 1. A 2. B 3. C 4. D 5.Digoxin may cause a type A adverse drug reaction due to: 1. Idiosyncratic effects 2. Its narrow therapeutic index 3. Being a teratogen 4. Being a carcinogen 6.Sarah developed a rash after using a topical medication. This is a type allergic drug reaction. 1. I 2. II 3. III 4. IV 7.A patient may develop neutropenia from using topical Silvadene for burns. Neutropenia is a(n): 1. Cytotoxic hypersensitivity reaction 2. Immune complex hypersensitivity 3. Immediate hypersensitivity reaction 4. Delayed hypersensitivity reaction 8.Anaphylactic shock is a: 1. Type I reaction, called immediate hypersensitivity reaction 2. Type II reaction, called cytotoxic hypersensitivity reaction 3. Type III allergic reaction, called immune complex hypersensitivity 4. Type IV allergic reaction, called delayed hypersensitivity reaction 9.James has hypothalamic-pituitary-adrenal axis suppression from chronic prednisone (a corticosteroid) use. He is at risk for what type of adverse drug reaction? 1. Type B 2. Type C 3. Type E 4. Type F 10. Immunomodulators such as azathioprine may cause a delayed adverse drug reaction known as a type D reaction because they are known: 1. Teratogens 2. Carcinogens 3. To cause hypersensitivity reactions 4. Hypothalamus-pituitary-adrenal axis suppressants 11. A 24-year-old male received multiple fractures in a motor vehicle accident that required significant amounts of opioid medication to treat his pain. He is at risk for a adverse drug reaction when he no longer requires the opioids. 1. Rapid 2. First-dose 3. Late 4. Delayed 12. An example of a first-dose reaction that may occur includes: 1. Orthostatic hypotension that does not occur with repeated doses 2. Purple glove syndrome with phenytoin use 3. Hemolytic anemia from ceftriaxone use 4. Contact dermatitis from neomycin use 13. Drugs that are prone to cause adverse drug effects include: 1. Diuretics 2. Inhaled anticholinergics 3. Insulins 4. Stimulants 14. The U.S. Food and Drug Administration MedWatch system is activated when: 1. There is an adverse event to a vaccine. 2. The patient has a severe reaction that is noted in the “Severe Reaction” section in the medication label. 3. A lactating woman takes a medication that is potentially toxic to the breastfeeding infant. 4. An adverse event or serious problem occurs with a medication that is not already identified on the label. 15. The Vaccine Adverse Events Reporting System is: 1. A mandatory reporting system for all health-care providers when they encounter an adverse vaccine event 2. A voluntary reporting system that health-care providers or consumers may use to report vaccine adverse events 3. Utilized to send out safety alerts regarding emerging vaccine safety issues 4. Activated when a vaccine has been proven to cause significant adverse effects Chapter 6. Factors That Foster Positive Outcomes 1.A comprehensive assessment of a patient should be holistic when trying to determine competence in drug administration. Which of the following factors would the NP omit from this type of assessment? 1. Financial status 2. Mobility 3. Social support 4. Sexual practices 2.Elena Vasquez’s primary language is Spanish, and she speaks very limited English. Which technique would be appropriate to use in teaching her about a new drug you have just prescribed? 1. Use correct medical terminology because Spanish has a Latin base. 2. Use a family member who speaks more English to act as an interpreter. 3. Use a professional interpreter or a reliable staff member who can act as an interpreter. 4. Use careful, detailed explanations. 3.Rod, age 68, has hearing difficulty. Which of the following would NOT be helpful in assuring that he understands teaching about his drug? 1. Stand facing him and speak slowly and clearly. 2. Speak in low tones or find a provider who has a lower voice. 3. Write down the instructions as well as speaking them. 4. If he reads lips, exaggerate lips movements when pronouncing the vowel sounds. 4.Which of the following factors may adversely affect a patient’s adherence to a therapeutic drug regimen? 1. Complexity of the drug regimen 2. Patient perception of the potential adverse effects of the drugs 3. Both 1 and 2 4. Neither 1 nor 2 5.The health-care delivery system itself can create barriers to adherence to a treatment regimen. Which of the following system variables creates such a barrier? 1. Increasing copayments for care 2. Unrestricted formularies for drugs, including brand names 3. Increasing the number of people who have access to care 4. Treating a wider range of disorders 6.Ralph’s blood pressure remains elevated despite increased doses of his drug. The NP is concerned that he might not be adhering to his treatment regimen. Which of the following events would suggest that he might not be adherent? 1. Ralph states that he always takes the drug “when I feel my pressure is going up.” 2. Ralph contacts his NP to discuss the need to increase the dosage. 3. Ralph consistently keeps his follow-up appointments to check his blood pressure. 4. All of the above show that he is adherent to the drug regimen. 7.Nonadherence is especially common in drugs that treat asymptomatic conditions, such as hypertension. One way to reduce the likelihood of nonadherence to these drugs is to prescribe a drug that: 1. Has a short half-life so that missing one dose has limited effect 2. Requires several dosage titrations so that missed doses can be replaced with lower doses to keep costs down 3. Has a tolerability profile with fewer of the adverse effects that are considered “irritating,” such as nausea and dizziness 4. Must be taken no more than twice a day 8.Factors in chronic conditions that contribute to nonadherence include: 1. The complexity of the treatment regimen 2. The length of time over which it must be taken 3. Breaks in the usual daily routine, such as vacations and weekends 4. All of the above 3. Inducing CYP 1A2, possibly leading to therapeutic failure of drugs metabolized by CYP 1A2 4. Decreasing first-pass metabolism of drugs 9.Milk and other foods that alkalinize the urine may: 1. Result in basic drugs being reabsorbed in the renal tubule 2. Increase the elimination of basic drugs in the urine 3. Decrease the elimination of acidic drugs 4. Not alter drug elimination due to the minimal change in urine pH 10.Antacids such as calcium carbonate (Tums) can reduce the absorption of which of the following nutrients? 1. Protein 2. Calcium 3. Iron 4. Vitamin K 11.Phenytoin decreases folic acid absorption by: 1. Altering the pH of the stomach 2. Increasing gastric emptying time 3. Inhibiting intestinal enzymes required for folic acid absorption 4. Chelation of the folic acid into inactive ingredients 12.Patients taking warfarin need to be educated about the vitamin K content of foods to avoid therapeutic failure. Foods high in vitamin K that should be limited to no more than one serving per day include: 1. Spinach 2. Milk 3. Romaine lettuce 4. Cauliflower 13.The American Dietetic Association has recommended the use of specific nutritional supplements in the following population(s): 1. 400 IU per day of vitamin D in all infants and children 2. 1,000 IU per day of vitamin D for all pregnant women 3. 60 mg per day of iron for all adults over age 50 years 4. All of the above 14.The American Dietetic Association recommends pregnant women take a supplement including: 1. 1,000 IU daily of vitamin D 2. 2.4 mcg/day of vitamin B12 3. 600 mcg/day of folic acid 4. 8 mg/day of iron 15.The American Heart Association and the American Dietetic Association recommend a minimum daily fiber intake of for cardiovascular health: 1. 10 mg/day 2. 15 mg/day 3. 20 mg/day 4. 25 mg/day 16.Which of the following vitamin or mineral supplements may by teratogenic if a pregnant woman takes more than the recommended amount? 1. Iron 2. Vitamin A 3. Vitamin B6 4. Vitamin C 17.Vitamin B2 (riboflavin) may be prescribed to: 1. Decrease the incidence of beriberi 2. Reduce headaches and migraines 3. Prevent pernicious anemia 4. Treat hyperlipidemia 18. Isoniazid (INH) may induce a deficiency of which vitamin? 1. B6 2. C 3. D 4. E 19.Pregnant patients who are taking isoniazid (INH) should take 25 mg/day of vitamin B6 (pyridoxine) to prevent: 1. Beriberi 2. Peripheral neuropathy 3. Rickets 4. Megaloblastic anemia 20.Vitamin B12 deficiency may lead to: 1. Hair loss 2. Insomnia 3. Dry scales on the scalp 4. Numbness and tingling of the hands 21.Smokers are at risk for vitamin C deficiency. It is recommended that smokers take vitamin C supplement. 1. 100 mg/day 2. 500 mg/day 3. 1,000 mg/day 4. 35 mg/day more than nonsmokers 22.There is strong evidence to support that adequate vitamin C intake prevents: 1. The common cold 2. Breast cancer 3. Scurvy 4. All of the above 23.Adequate vitamin D is needed for: 1. Absorption of calcium from the gastrointestinal tract 2. Regulation of serum calcium levels 3. Regulation of serum phosphate levels 4. All of the above 24.Newborns are at risk for early vitamin K deficiency bleeding and the American Academy of Pediatrics recommends that all newborns receive: 1. IM vitamin K (phytonadione) within 24 hours of birth 2. Oral vitamin K supplementation in the first 3 weeks of life 3. Formula containing vitamin K or breast milk 4. Oral vitamin K in the first 24 hours after birth 25.Symptoms of folate deficiency include: 1. Thinning of the hair 2. Bruising easily 3. Glossitis 4. Numbness and tingling of the hands and feet 26.A patient with a new onset of systolic ejection murmur should be assessed for which nutritional deficiency? 1. Vitamin B12 2. Vitamin C 3. Folate 4. Niacine 27.According to the 2003-2006 National Health and Nutrition Examination Survey study of dietary intake, the group at highest risk for inadequate calcium intake was: 1. The elderly (over age 60 years) 2. Teenage females 3. Teenage males 4. Preschoolers 28.Patients with iron deficiency will develop: 1. Hemolytic anemia 2. Megaloblastic anemia 3. Macrocytic-hypochromic anemia 4. Microcytic-hypochromic anemia 29.There is evidence that dietary supplementation or adequate intake of fish oils and omega-3 fatty acids have well-documented: 1. Concern for developing cardiac dysrhythmias 2. Anti-inflammatory effects 3. Total cholesterol-lowering effects 4. Effects on fasting blood sugar 30.There is enough preliminary evidence to recommend that children with autism receive which supplemental nutrient? 1. Vitamin B1 (thiamine) 2. Vitamin B2 (riboflavin) 3. Calcium 3. Lowers supine blood pressure more than standing pressure 4. Relaxes smooth muscle in the bladder neck 2.To reduce potential adverse effects, patients taking a peripherally acting alpha1 antagonist should do all of the following EXCEPT: 1. Take the dose at bedtime 2. Sit up slowly and dangle their feet before standing 3. Monitor their blood pressure and skip a dose if the pressure is less than 120/80 4. Weigh daily and report weight gain of greater than 2 pounds in one day 3.John has clonidine, a centrally acting adrenergic blocker, prescribed for his hypertension. He should: 1. Not miss a dose or stop taking the drug because of potential rebound hypertension 2. Increase fiber in the diet to treat any diarrhea that may occur 3. Reduce fluid intake to less than 2 liters per day to prevent fluid retention 4. Avoid sitting for long periods, as this can lead to deep vein thrombosis 4.Clonidine has several off-label uses, including: 1. Alcohol and nicotine withdrawal 2. Post-herpetic neuralgia 3. Both 1 and 2 4. Neither 1 nor 2 5.Jim is being treated for hypertension. Because he has a history of heart attack, the drug chosen is atenolol. Beta blockers treat hypertension by: 1. Increasing heart rate to improve cardiac output 2. Reducing vascular smooth muscle tone 3. Increasing aldosterone-mediated volume activity 4. Reducing aqueous humor production 6.Which of the following adverse effects are less likely in a beta1-selective blocker? 1. Dysrhythmias 2. Impaired insulin release 3. Reflex orthostatic changes 4. Decreased triglycerides and cholesterol 7.Richard is 70 years old and has a history of cardiac dysrhythmias. He has been prescribed nadolol. You do his annual laboratory work and find a CrCl of 25 ml/min. What action should you take related to his nadolol? 1. Extend the dosage interval. 2. Decrease the dose by 75%. 3. Take no action because this value is expected in the older adult. 4. Schedule a serum creatinine level to validate the CrCl value. 8.Beta blockers are the drugs of choice for exertional angina because they: 1. Improve myocardial oxygen supply by vasodilating the coronary arteries 2. Decrease myocardial oxygen demand by decreasing heart rate and vascular resistance 3. Both 1 and 2 4. Neither 1 nor 2 9.Adherence to beta blocker therapy may be affected by their: 1. Short half-lives requiring twice daily dosing 2. Tendency to elevate lipid levels 3. Effects on the male genitalia, which may produce impotence 4. None of the above 10.Beta blockers have favorable effects on survival and disease progression in heart failure. Treatment should be initiated when the: 1. Symptoms are severe 2. Patient has not responded to other therapies 3. Patient has concurrent hypertension 4. Left ventricular dysfunction is diagnosed 11.Abrupt withdrawal of beta blockers can be life threatening. Patients at highest risk for serious consequences of rapid withdrawal are those with: 1. Angina 2. Coronary artery disease 3. Both 1 and 2 4. Neither 1 nor 2 12.To prevent life-threatening events from rapid withdrawal of a beta blocker: 1. The dosage interval should be increased by 1 hour each day. 2. An alpha blocker should be added to the treatment regimen before withdrawal. 3. The dosage should be tapered over a period of weeks. 4. The dosage should be decreased by one-half every 4 days. 13.Beta blockers are prescribed for diabetics with caution because of their ability to produce hypoglycemia and block the common symptoms of it. Which of the following symptoms of hypoglycemia is not blocked by these drugs and so can be used to warn diabetics of possible decreased blood glucose? 1. Dizziness 2. Increased heart rate 3. Nervousness and shakiness 4. Diaphoresis 14.Combined alpha-beta antagonists are used to reduce the progression of heart failure because they: 1. Vasodilate the peripheral vasculature 2. Decrease cardiac output 3. Increase renal vascular resistance 4. Reduce atherosclerosis secondary to elevated serum lipoproteins 15.Carvedilol is heavily metabolized by CYP2D6 and 2C9, resulting in drug interactions with which of the following drug classes? 1. Histamine 2 blockers 2. Quinolones 3. Serotonin re-uptake inhibitors 4. All of the above 16.Alpha-beta blockers are especially effective to treat hypertension for which ethnic group? 1. White 2. Asian 3. African American 4. Native American 17.Bethanechol: 1. Increases detrusor muscle tone to empty the bladder 2. Decreases gastric acid secretion to treat peptic ulcer disease 3. Stimulates voluntary muscle tone to improve strength 4. Reduces bronchial airway constriction to treat asthma 18.Clinical dosing of Bethanechol: 1. Begins at the highest effective dose to obtain a rapid response 2. Starts at 5 mg to 10 mg PO and is repeated every hour until a satisfactory clinical response is achieved 3. Requires dosing only once daily 4. Is the same for both the oral and parenteral route 19.Patients who need to remain alert are taught to avoid which drug due to its antimuscarinic effects? 1. Levothyroxine 2. Prilosec 3. Dulcolax 4. Diphenhydramine 20.Anticholinesterase inhibitors are used to treat: 1. Peptic ulcer disease 2. Myasthenia gravis 3. Both 1 and 2 4. Neither 1 nor 2 21.Which of the following drugs used to treat Alzheimer’s disease is not an anticholinergic? 1. Donepezil 2. Memantine 3. Rivastigmine 4. Galantamine 22.Taking which drug with food maximizes it bioavailability? 1. Donepezil 2. Galantamine 3. Rivastigmine 4. Memantine 23.Which of the following drugs should be used only when clearly needed in pregnant and breastfeeding women? 1. Memantine 2. Pyridostigmine 3. Cardiovascular 4. Respiratory Chapter 15. Drugs Affecting the Central Nervous System 1.Sarah, a 42-year-old female, requests a prescription for an anorexiant to treat her obesity. A trial of phentermine is prescribed. Prescribing precautions include: 1. Understanding that obesity is a contraindication to prescribing phentermine 2. Anorexiants may cause tolerance and should only be prescribed for 6 months 3. Patients should be monitored for postural hypotension 4. Renal function should be monitored closely while on anorexiants 2.Before prescribing phentermine to Sarah, a thorough drug history should be taken including assessing for the use of serotonergic agents such as selective serotonin reuptake inhibitors (SSRIs) and St John’s wort due to: 1. Additive respiratory depression risk 2. Additive effects affecting liver function 3. The risk of serotonin syndrome 4. The risk of altered cognitive functioning 3.Antonia is a 3-year-old child who has a history of status epilepticus. Along with her routine antiseizure medication, she should also have a home prescription for to be used for an episode of status epilepticus. 1. IV phenobarbital 2. Rectal diazepam (Diastat) 3. IV phenytoin (Dilantin) 4. Oral carbamazepine (Tegretol) 4.Rabi is being prescribed phenytoin for seizures. Monitoring includes assessing: 1. For phenytoin hypersensitivity syndrome 3 to 8 weeks after starting treatment 2. For pedal edema throughout therapy 3. Heart rate at each visit and consider altering therapy if heart rate is less than 60 bpm 4. For vision changes, such as red-green blindness, at least annually 5.Dwayne has recently started on carbamazepine to treat seizures. He comes to see you and you note that while his carbamazepine levels had been in the therapeutic range, they are now low. The possible cause for the low carbamazepine levels include: 1. Dwayne hasn’t been taking his carbamazepine because it causes insomnia. 2. Carbamazepine auto-induces metabolism, leading to lower levels in spite of good compliance. 3. Dwayne was not originally prescribed the correct amount of carbamazepine. 4. Carbamazepine is probably not the right antiseizure medication for Dwayne. 6.Carbamazepine has a Black Box Warning due to life-threatening: 1. Renal toxicity, leading to renal failure 2. Hepatotoxicity, leading to liver failure 3. Dermatologic reaction, including Steven’s Johnson and toxic epidermal necrolysis 4. Cardiac effects, including supraventricular tachycardia 7.Long-term monitoring of patients who are taking carbamazepine includes: 1. Routine troponin levels to assess for cardiac damage 2. Annual eye examinations to assess for cataract development 3. Monthly pregnancy tests for all women of childbearing age 4. Complete blood count every 3 to 4 months 8.Six-year-old Lucy has recently been started on ethosuximide (Zarontin) for seizures. She should be monitored for: 1. Increased seizure activity, as this drug may auto-induce seizures 2. Altered renal function, including renal failure 3. Blood dyscrasias, which are uncommon but possible 4. Central nervous system excitement, leading to insomnia 9.Sook has been prescribed gabapentin to treat neuropathic pain and is complaining of feeling depressed and having “strange” thoughts. The appropriate initial action would be: 1. Increase her dose 2. Assess for suicidal ideation 3. Discontinue the medication immediately 4. Decrease her dose to half then slowly titrate up the dose 10.Selma, who is overweight, recently started taking topiramate for seizures and at her follow-up visit you note she has lost 3 kg. The appropriate action would be: 1. Tell her to increase her caloric intake to counter the effects of the topiramate. 2. Consult with a neurologist, as this is not a common adverse effect of topiramate. 3. Decrease her dose of topiramate. 4. Reassure her that this is a normal side effect of topiramate and continue to monitor her weight. 11.Monitoring of a patient on gabapentin to treat seizures includes: 1. Routine therapeutic drug levels every 3 to 4 months 2. Assessing for dermatologic reactions, including Steven’s Johnson 3. Routine serum electrolytes, especially in hot weather 4. Recording seizure frequency, duration, and severity 12.Scott’s seizures are well controlled on topiramate and he wants to start playing baseball. Education for Scott regarding his topiramate includes: 1. He should not play sports due to the risk of increased seizures 2. He should monitor his temperature and ability to sweat in the heat while playing 3. Reminding him that he may need higher dosages of topiramate when exercising 4. Encouraging him to use sunscreen due to photosensitivity from topiramate 13.Cara is taking levetiracetam (Keppra) to treat seizures. Routine education for levetiracetam includes reminding her: 1. To not abruptly discontinue levetiracetam due to risk for withdrawal seizures 2. To wear sunscreen due to photosensitivity from levetiracetam 3. To get an annual eye exam while on levetiracetam 4. To report weight loss if it occurs 14.Levetiracetam has known drug interactions with: 1. Combined oral contraceptives 2. Carbamazepine 3. Warfarin 4. Few, if any, drugs 15.Zainab is taking lamotrigine (Lamictal) and presents to the clinic with fever and lymphadenopathy. Initial evaluation and treatment includes: 1. Reassuring her she has a viral infection and to call if she isn’t better in 4 or 5 days 2. Ruling out a hypersensitivity reaction that may lead to multi-organ failure 3. Rapid strep test and symptomatic care if strep test is negative 4. Observation only, with further assessment if she worsens 16.Samantha is taking lamotrigine (Lamictal) for her seizures and requests a prescription for combined oral contraceptives (COCs), which interact with lamotrigine and may cause: 1. Contraceptive failure 2. Excessive weight gain 3. Reduced lamotrigine levels, requiring doubling the dose of lamotrigine 4. Induction of estrogen metabolism, requiring higher estrogen content OCs be prescribed 17.The tricyclic antidepressants should be prescribed cautiously in patients with: 1. Eczema 2. Asthma 3. Diabetes 4. Heart disease 18.A 66-year-old male was prescribed phenelzine (Nardil) while in an acute psychiatric unit for recalcitrant depression. The NP managing his primary health care needs to understand the following regarding phenelzine and other monoamine oxidase inhibitors (MAOIs): 1. He should not be prescribed any serotonergic drug such as sumatriptan (Imitrex) 2. MAOIs interact with many common foods, including yogurt, sour cream, and soy sauce 3. Symptoms of hypertensive crisis (headache, tachycardia, sweating) require immediate treatment 4. All of the above 19.Taylor is a 10-year-old child diagnosed with major depression. The appropriate first-line antidepressant for children is: 1. Fluoxetine 2. Fluvoxamine 3. Sertraline 4. Escitalopram 20.Suzanne is started on paroxetine (Paxil), a selective serotonin reuptake inhibitor (SSRI), for depression. Education regarding her antidepressant includes: 1. SSRIs may take 2 to 6 weeks before she will have maximum drug effects. same time. 4. All of the above 34.Kirk sprained his ankle and is asking for pain medication for his mild-to-moderate pain. The appropriate first-line medication would be: 1. Ibuprofen (Advil) 2. Acetaminophen with hydrocodone (Vicodin) 3. Oxycodone (Oxycontin) 4. Oral morphine (Roxanol) 35.Kasey fractured his ankle in two places and is asking for medication for his pain. The appropriate first- line medication would be: 1. Ibuprofen (Advil) 2. Acetaminophen with hydrocodone (Vicodin) 3. Oxycodone (Oxycontin) 4. Oral morphine (Roxanol) 36.Jack, age 8, has attention deficit disorder (ADD) and is prescribed methylphenidate (Ritalin). He and his parents should be educated about the side effects of methylphenidate, which are: 1. Slurred speech and insomnia 2. Bradycardia and confusion 3. Dizziness and orthostatic hypotension 4. Insomnia and decreased appetite 37.Monitoring for a child on methylphenidate for attention deficit hyperactivity disorder (ADHD) includes: 1. ADHD symptoms 2. Routine height and weight checks 3. Amount of methylphenidate being used 4. All of the above 38.When prescribing Adderall (amphetamine and dextroamphetamine) to adults with ADHD the nurse practitioner will need to monitor: 1. Blood pressure 2. Blood glucose levels 3. Urine ketone levels 4. Liver function Chapter 16. Drugs Affecting the Cardiovascular and Renal Systems 1. Ray has been diagnosed with hypertension and an angiotensin-converting enzyme inhibitor is determined to be needed. Prior to prescribing this drug, the NP should assess for: 1. Hypokalemia 2. Impotence 3. Decreased renal function 4. Inability to concentrate 2. Angiotensin-converting enzyme inhibitors are the drug of choice in treating hypertension in diabetic patients because they: 1. Improve insulin sensitivity 2. Improve renal hemodynamics 3. Reduce the production of angiotensin II 4. All of the above 3. A potentially life-threatening adverse response to angiotensin-converting enzyme inhibitors is angioedema. Which of the following statements is true about this adverse response? 1. Swelling of the tongue or hoarseness are the most common symptoms. 2. It appears to be related to the decrease in aldosterone production. 3. Presence of a dry, hacky cough indicates a high risk for this adverse response. 4. Because it takes time to build up a blood level, it occurs after being on the drug for about 1 week. 4. Angiotensin-converting enzyme inhibitors are useful in a variety of disorders. Which of the following statements are true about both its usefulness in the disorder and the reason for its use? 1. Stable angina because it decreases the thickening of vascular walls due to decreased modified release. 2. Heart failure because it reduces remodeling of injured myocardial tissues. 3. Both 1 and 2 are true and the reasons are correct. 4. Both 1 and 2 are true but the reasons are wrong. 5. Neither 1 nor 2 are true. 5. Despite good blood pressure control, an NP might change a patient’s drug from an angiotensin- converting enzyme (ACE) inhibitor to an angiotensin II receptor blocker (ARB) because the ARB: 1. Is stronger than the ACE inhibitor 2. Does not produce a dry, hacky cough 3. Has no effect on the renal system 4. Reduces sodium and water retention 6. While taking an angiotensin II receptor blocker (ARB), patients need to avoid certain over-the- counter drugs without first consulting the provider because: 1. Cimetidine is metabolized by the CYP 3A4 isoenzymes 2. Nonsteroidal anti-inflammatory drugs reduce prostaglandin levels 3. Both 1 and 2 4. Neither 1 nor 2 7. Laboratory monitoring for patients on angiotensin-converting enzyme inhibitors or angiotensin II receptor blockers should include: 1. White blood cell counts with the drug dosage increased for elevations above 10,000 feet 2. Liver function tests with the drug dosage stopped for alanine aminotransferase values twice that of normal 3. Serum creatinine levels with the drug dosage reduced for values greater than 2.5 mg/dL 4. Serum glucose levels with the drug dosage increased for levels greater than 120 mg/dL 8. Jacob has hypertension, for which a calcium channel blocker has been prescribed. This drug helps control blood pressure because it: 1. Decreases the amount of calcium inside the cell 2. Reduces stroke volume 3. Increases the activity of the Na+/K+/ATPase pump indirectly 4. Decreases heart rate 9. Which of the following adverse effects may occur due to a dihydropyridine-type calcium channel blocker? 1. Bradycardia 2. Hepatic impairment 3. Increased contractility 4. Edema of the hands and feet 10. Patient teaching related to amlodipine includes: 1. Increase calcium intake to prevent osteoporosis from a calcium blockade. 2. Do not crush the tablet; it must be given in liquid form if the patient has trouble swallowing it. 3. Avoid grapefruit juice as it affects the metabolism of this drug. 4. Rise slowly from a supine position to reduce orthostatic hypotension. 11. Vera, age 70, has isolated systolic hypertension. Calcium channel blocker dosages for her should be: 1. Started at about half the usual dosage 2. Not increased over the usual dosage for an adult 3. Given once daily because of memory issues in the older adult 4. Withheld if she experiences gastroesophageal reflux 12. Larry has heart failure, which is being treated with digoxin because it exhibits: 1. Negative inotropism 2. Positive chronotropism 3. Both 1 and 2 4. Neither 1 nor 2 13. Furosemide is added to a treatment regimen for heart failure that includes digoxin. Monitoring for this combination includes: 1. Hemoglobin 2. Serum potassium 3. Blood urea nitrogen 4. Serum glucose 14. Which of the following create a higher risk for digoxin toxicity? Both the cause and the reason for it must be correct. 1. Older adults because of reduced renal function 2. Administration of aldosterone antagonist diuretics because of decreased potassium levels 3. Taking an antacid for gastroesophageal reflux disease because it increases the absorption of digoxin 4. Headaches 28. Dulcea has type 2 diabetes and a high triglyceride level. She has gemfibrozil prescribed to treat her hypertriglyceridemia. A history of which of the following might contraindicate the use of this drug? 1. Reactive airway disease/asthma 2. Inflammatory bowel disease 3. Allergy to aspirin 4. Gallbladder disease 29. Many patients with hyperlipidemia are treated with more than one drug. Combining a fibric acid derivative such as gemfibrozil with which of the following is not recommended? The drug and the reason must both be correct for the answer to be correct. 1. Reductase inhibitors, due to an increased risk for rhabdomyolysis 2. Bile-acid sequestering resins, due to interference with folic acid absorption 3. Grapefruit juice, due to interference with metabolism 4. Niacin, due to decreased gemfibrozil activity 30. Felicity has been prescribed colestipol to treat her hyperlipidemia. Unlike other anti-lipidemics, this drug: 1. Blocks synthesis of cholesterol in the liver 2. Exchanges chloride ions for negatively charged acids in the bowel 3. Increases HDL levels the most among the classes 4. Blocks the lipoprotein lipase pathway 31. Because of their site of action, bile acid sequestering resins: 1. Should be administered separately from other drugs by at least 4 hours 2. May increase the risk for bleeding 3. Both 1 and 2 4. Neither 1 nor 2 32. Colestipol comes in a powdered form. The patient is taught to: 1. Take the powder dry and follow it with at least 8 ounces of water 2. Take it with a meal to enhance its action on fatty food 3. Mix the powder with 4 to 6 ounces of milk or fruit juice 4. Take after the evening meal to coincide with cholesterol synthesis 33. The choice of diuretic to use in treating hypertension is based on: 1. Presence of diabetes with loop diuretics being used for these patients 2. Level of kidney function with a thiazide diuretic being used for an estimated glomerular filtration rate higher than the mid-40mL/min range 3. Ethnicity with aldosterone antagonists best for African Americans and older adults 4. Presence of hyperlipidemia with higher doses needed for patients with LDL above 130 mg/dL 34. Direct renin inhibitors have the following properties. They: 1. Are primarily generic drugs 2. Are a renin-angiotensin-aldosterone system (RAAS) medication that is safe during pregnancy 3. Can be used with an angiotensin-converting enzyme and angiotensin II receptor blocker medications for stronger impact 4. “Shut down” the entire RAAS cycle 35. When comparing angiotensin-converting enzyme (ACE) and angiotensin II receptor blocker (ARB) medications, which of the following holds true? 1. Both have major issues with a dry, irritating cough 2. Both contribute to some retention of potassium 3. ARBs have a stronger impact on hypertension control than ACE medications 4. ARBs have stronger diabetes mellitus renal protection properties than ACE medications 36. What does the provider understand about the issue of “Diabetic Renal Protection” with angiotensin-converting enzyme (ACE) medications? Diabetes mellitus patients: 1. Have a reduced rate of renal progression, but still need to be discontinued when advanced renal issues present 2. Who start these medications never progress to renal nephropathy 3. With early renal dysfunction will see it reverse when on ACE medications 4. Without renal issues are the only ones who benefit from ACE protection 37. What dermatological issue is linked to Amiodarone use? 1. Increased risk of basal cell carcinoma 2. Flare up of any prior psoriasis problems 3. Development of plantar warts 4. Progressive change of skin tone toward a blue spectrum 38. Commercials on TV for erectile dysfunction (ED) medications warn about mixing them with nitrates. Why? 1. Increased risk of priapism 2. Profound hypotension 3. Development of blue discoloration to the visual field 4. Inactivation of the ED medication effect Chapter 17. Drugs Affecting the Respiratory System 1. Digoxin levels need to be monitored closely when the following medication is started: 1. Loratadine 2. Diphenhydramine 3. Ipratropium 4. Albuterol 2. Patients with pheochromocytoma should avoid which of the following classes of drugs because of the possibility of developing hypertensive crisis? 1. Expectorants 2. Beta-2-agonists 3. Antitussives 4. Antihistamines 3. Harold, a 42-year-old African American, has moderate persistent asthma. Which of the following asthma medications should be used cautiously, if at all? 1. Betamethasone, an inhaled corticosteroid 2. Salmeterol, an inhaled long-acting beta-agonist 3. Albuterol, a short-acting beta-agonist 4. Montelukast, a leukotriene modifier 4. Long-acting beta-agonists (LTBAs) received a Black Box Warning from the U.S. Food and Drug Administration due to the: 1. Risk of life-threatening dermatological reactions 2. Increased incidence of cardiac events when LTBAs are used 3. Increased risk of asthma-related deaths when LTBAs are used 4. Risk for life-threatening alterations in electrolytes 5. The bronchodilator of choice for patients taking propranolol is: 1. Albuterol 2. Pirbuterol 3. Formoterol 4. Ipratropium 6. James is a 52-year-old overweight smoker taking theophylline for his persistent asthma. He tells his provider he is going to start the Atkin’s diet for weight loss. The appropriate response would be: 1. Congratulate him on making a positive change in his life. 2. Recommend he try stopping smoking instead of the Atkin’s diet. 3. Schedule him for regular testing of serum theophylline levels during his diet due to increased excretion of theophylline. 4. Decrease his theophylline dose because a high-protein diet may lead to elevated theophylline levels. 7. Li takes theophylline for his persistent asthma and calls the office with a complaint of nausea, vomiting, and headache. The best advice for him would be to: 1. Reassure him this is probably a viral infection and should be better soon 2. Have him seen the same day for an assessment and theophylline level 3. Schedule him for an appointment in 2 to 3 days, which he can cancel if he is better 4. Order a theophylline level at the laboratory for him 8. Tiotropium bromide (Spiriva) is an inhaled anticholinergic: 1. Used for the treatment of chronic obstructive pulmonary disease (COPD) 2. Used in the treatment of asthma 3. Combined with albuterol for treatment of asthma exacerbations 4. Combined with fluticasone for the treatment of persistent asthma 9. Christy has exercise-induced and mild persistent asthma and is prescribed two puffs of albuterol 15 minutes before exercise and as needed for wheezing. One puff per day of beclomethasone (QVAR) is also prescribed. Teaching regarding her inhalers includes: 1. Use one to two puffs of albuterol per day to prevent an attack with no more than 2. Juanita had a deep vein thrombosis (DVT) and was on heparin in the hospital and was discharged on warfarin. She asks her primary care provider NP why she was getting both medications while in the hospital. The best response is to: 1. Contact the hospitalist as this is not the normal guideline for prescribing these two medications and she may have had a more complicated case. 2. Explain that warfarin is often started while a patient is still on heparin because warfarin takes a few days to reach effectiveness. 3. Encourage the patient to contact the Customer Service department at the hospital as this was most likely a medication error during her admission. 4. Draw anticoagulation studies to make sure she does not have dangerously high bleeding times. 3. The safest drug to use to treat pregnant women who require anticoagulant therapy is: 1. Low-molecular-weight heparin 2. Warfarin 3. Aspirin 4. Heparin 4. The average starting dose of warfarin is 5 mg daily. Higher doses of 7.5 mg daily should be considered in which patients? 1. Pregnant women 2. Elderly men 3. Overweight or obese patients 4. Patients with multiple comorbidities 5. Cecil and his wife are traveling to Southeast Asia on vacation and he has come into the clinic to review his medications. He is healthy with only mild hypertension that is well controlled. He asks about getting “a shot” to prevent blood clots like his friend Ralph did before international travel. The correct respond would be: 1. Administer one dose of low-molecular weight heparin 24 hours before travel. 2. Prescribe one dose of warfarin to be taken the day of travel. 3. Consult with a hematologist regarding a treatment plan for Cecil. 4. Explain that Cecil is not at high risk of a blood clot and provide education about how to prevent blood clots while traveling. 6. Robert, age 51 years, has been told by his primary care provider (PCP) to take an aspirin a day. Why would this be recommended? 1. He has arthritis and this will help with the inflammation and pain. 2. Aspirin has anti-platelet activity and prevents clots that cause heart attacks. 3. Aspirin acidifies the urine and he needs this for prostrate health. 4. He has a history of GI bleed, and one aspirin a day is a safe dosage. 7. Sally has been prescribed aspirin 320 mg per day for her atrial fibrillation. She also takes aspirin four or more times a day for arthritis pain. What are the symptoms of aspirin toxicity for which she would need to be evaluated? 1. Tinnitus 2. Diarrhea 3. Hearing loss 4. Photosensitivity 8. Patient education when prescribing clopidogrel includes: 1. Do not take any herbal products without discussing it with the provider. 2. Monitor urine output closely and contact the provider if it decreases. 3. Clopidogrel can be constipating, use a stool softener if needed. 4. The patient will need regular anticoagulant studies while on clopidogrel. 9. For patients taking warfarin, INRs are best drawn: 1. Monthly throughout therapy 2. Three times a week throughout therapy 3. Two hours after the last dose of warfarin to get an accurate peak level 4. In the morning if the patient takes their warfarin at night 10. Patients receiving heparin therapy require monitoring of: 1. Platelets every 2 to 3 days for thrombocytopenia that may occur on day 4 of therapy 2. Electrolytes for elevated potassium levels in the first 24 hours of therapy 3. INR throughout therapy to stay within the range of 2.0 4. Blood pressure for hypertension that may occur in the first 2 days of treatment 11. The routine monitoring recommended for low molecular weight heparin is: 1. INR every 2 days until stable, then weekly 2. aPTT every week while on therapy 3. Factor Xa levels if the patient is pregnant 4. White blood cell count every 2 weeks 12. When writing a prescription for warfarin it is common to write on the prescription. 1. OK to substitute for generic 2. The brand name of warfarin and Do Not Substitute 3. PRN refills 4. Refills for 1 year 13. Education of patients who are taking warfarin includes discussing their diet. Instructions include: 1. Avoiding all vitamin K-containing foods 2. Avoiding high-vitamin K-containing foods 3. Increasing intake of iron-containing foods 4. Making sure they eat 35 grams of fiber daily 14. Patients who are being treated with epoetin alfa need to be monitored for the development of: 1. Thrombocytopenia 2. Neutropenia 3. Hypertension 4. Gout 15. The FDA issued a safety announcement regarding the use of erythropoiesis-stimulating agents (ESAs) in 2010 with the recommendation that: 1. ESAs no longer be prescribed to patients with chronic renal failure 2. The risk of tumor development be explained to cancer patients on ESA therapy 3. Patients should no longer receive ESA therapy to prepare for allogenic transfusions 4. ESAs be prescribed only to patients younger than age 60 years 16. When patients are started on darbepoetin alfa (Aranesp) they need monitoring of their blood counts to determine a dosage adjustment in: 1. 6 weeks if they are a cancer patient 2. 1 week if they have chronic renal failure 3. 2 weeks if they are taking it for allogenic transfusion 4. Each week throughout therapy 17. Jim is having a hip replacement surgery and would like to self-donate blood for the surgery. In addition to being prescribed epogen alpha he should also be prescribed: 1. Folic acid to prevent megaloblastic anemia 2. Iron, to start when the epogen starts 3. An antihypertensive to counter the adverse effects of epogen 4. Vitamin B12 to prevent pernicious anemia 18. Monitoring for a patient being prescribed iron for iron deficiency anemia includes: 1. Reticulocyte count 1 week after therapy is started 2. Complete blood count every 2 weeks throughout therapy 3. Hemoglobin level at 1 week of therapy 4. INR weekly throughout therapy 19. Patient education regarding taking iron replacements includes: 1. Doubling the dose if they miss a dose to maintain therapeutic levels 2. Taking the iron with milk or crackers if it upsets their stomach 3. Iron is best taken on an empty stomach with juice 4. Antacids such as Tums may help the upset stomach caused by iron therapy 20. Patients with pernicious anemia require treatment with: 1. Iron 2. Folic acid 3. Epogen alpha 4. Vitamin B12 21. The first laboratory value indication that vitamin B12 therapy is adequately treating pernicious anemia is: 1. Hematocrit levels start to rise 2. Hemoglobin levels return to normal 3. Reticulocyte count begins to rise 4. Vitamin B12 levels return to normal 22. Patients who are beginning therapy with vitamin B12 need to be monitored for: 1. Hypertensive crisis that may occur in the first 36 hours 2. Hypokalemia that occurs in the first 48 hours 3. Leukopenia that occurs at 1 to 3 weeks of therapy 4. Thrombocytopenia that may occur at any time in therapy Chapter 20. Drugs Affecting the Gastrointestinal System 4. Pelvic bones 2. Bisphosphonates treat or prevent osteoporosis by: 1. Inhibiting osteoclastic activity 2. Fostering bone resorption 3. Enhancing calcium uptake in the bone 4. Strengthening the osteoclastic proton pump 3. Prophylactic use of bisphosphonates is recommended for patients with early osteopenia related to long-term use of which of the following drugs? 1. Selective estrogen receptor modulators 2. Aspirin 3. Glucocorticoids 4. Calcium supplements 4. Patients with cystic fibrosis are often prescribed enzyme replacement for pancreatic secretions. Each replacement drug has lipase, protease, and amylase components, but the drug is prescribed in units of: 1. Lipase 2. Protease 3. Amylase 4. Pancreatin 5. Brands of pancreatic enzyme replacement drugs are: 1. Bioequivalent 2. About the same in cost per unit of lipase across brands 3. Able to be interchanged between generic and brand-name products to reduce cost 4. None of the above 6. When given subcutaneously, how long until neutral protamine Hagedorn insulin begins to take effect (onset of action) after administration? 1. 15 to 30 minutes 2. 60 to 90 minutes 3. 3 to 4 hours 4. 6 to 8 hours 7. Hypoglycemia can result from the action of either insulin or an oral hypoglycemic. Signs and symptoms of hypoglycemia include: 1. “Fruity” breath odor and rapid respiration 2. Diarrhea, abdominal pain, weight loss, and hypertension 3. Dizziness, confusion, diaphoresis, and tachycardia 4. Easy bruising, palpitations, cardiac dysrhythmias, and coma 8. Nonselective beta blockers and alcohol create serious drug interactions with insulin because they: 1. Increase blood glucose levels 2. Produce unexplained diaphoresis 3. Interfere with the ability of the body to metabolize glucose 4. Mask the signs and symptoms of altered glucose levels 9. Lispro is an insulin analogue produced by recombinant DNA technology. Which of the following statements about this form of insulin is NOT true? 1. Optimal time of preprandial injection is 15 minutes. 2. Duration of action is increased when the dose is increased. 3. It is compatible with neutral protamine Hagedorn insulin. 4. It has no pronounced peak. 10. The decision may be made to switch from twice daily neutral protamine Hagedorn (NPH) insulin to insulin glargine to improve glycemia control throughout the day. If this is done: 1. The initial dose of glargine is reduced by 20% to avoid hypoglycemia. 2. The initial dose of glargine is 2 to 10 units per day. 3. Patients who have been on high doses of NPH will need tests for insulin antibodies. 4. Obese patients may require more than 100 units per day. 11. When blood glucose levels are difficult to control in type 2 diabetes some form of insulin may be added to the treatment regimen to control blood glucose and limit complication risks. Which of the following statements is accurate based on research? 1. Premixed insulin analogues are better at lowering HbA1C and have less risk for hypoglycemia. 2. Premixed insulin analogues and the newer premixed insulins are associated with more weight gain than the oral antidiabetic agents. 3. Newer premixed insulins are better at lowering HbA1C and postprandial glucose levels than long-acting insulins. 4. Patients who are not controlled on oral agents and have postprandial hyperglycemia can have neutral protamine Hagedorn insulin added at bedtime. 12. Metformin is a primary choice of drug to treat hyperglycemia in type 2 diabetes because it: 1. Substitutes for insulin usually secreted by the pancreas 2. Decreases glycogenolysis by the liver 3. Increases the release of insulin from beta cells 4. Decreases peripheral glucose utilization 13. Prior to prescribing metformin, the provider should: 1. Draw a serum creatinine to assess renal function 2. Try the patient on insulin 3. Tell the patient to increase iodine intake 4. Have the patient stop taking any sulfonylurea to avoid dangerous drug interactions 14. The action of “gliptins” is different from other antidiabetic agents because they: 1. Have a low risk for hypoglycemia 2. Are not associated with weight gain 3. Close ATP-dependent potassium channels in the beta cell 4. Act on the incretin system to indirectly increase insulin production 15. Sitagliptin has been approved for: 1. Monotherapy in once-daily doses 2. Combination therapy with metformin 3. Both 1 and 2 4. Neither 1 nor 2 16. GLP-1 agonists: 1. Directly bind to a receptor in the pancreatic beta cell 2. Have been approved for monotherapy 3. Speed gastric emptying to decrease appetite 4. Can be given orally once daily 17. Avoid concurrent administration of exenatide with which of the following drugs? 1. Digoxin 2. Warfarin 3. Lovastatin 4. All of the above 18. Administration of exenatide is by subcutaneous injection: 1. 30 minutes prior to the morning meal 2. 60 minutes prior to the morning and evening meal 3. 15 minutes after the evening meal 4. 60 minutes before each meal daily 19. Potentially fatal granulocytopenia has been associated with treatment of hyperthyroidism with propylthiouracil. Patients should be taught to report: 1. Tinnitus and decreased salivation 2. Fever and sore throat 3. Hypocalcemia and osteoporosis 4. Laryngeal edema and difficulty swallowing 20. Elderly patients who are started on levothyroxine for thyroid replacement should be monitored for: 1. Excessive sedation 2. Tachycardia and angina 3. Weight gain 4. Cold intolerance 21. Which of the following is not an indication that growth hormone supplements should be discontinued? 1. Imaging indication of epiphyseal closure 2. Growth curve increases have plateaued 3. Complaints of mild bone pain 4. Achievement of anticipated height goals 22. Besides osteoporosis, IV bisphosphonates are also indicated for: 1. Paget’s Disease 2. Early osteopenia 3. Renal cancer 4. Early closure of cranial sutures 23. What is the role of calcium supplements when patients take bisphosphonates? of the current pill pack. 4. If she gets nauseated with the medications she should call the office for an antiemetic prescription. 12. A 56-year-old woman is complaining of vaginal dryness and dyspareunia. To treat her symptoms with the lowest adverse effects she should be prescribed: 1. Low-dose oral estrogen 2. A low-dose estrogen/progesterone combination 3. A vaginal estradiol ring 4. Vaginal progesterone cream 13. Shana is receiving her first medroxyprogesterone (Depo Provera) injection. Shana will need to be monitored for: 1. Depression 2. Hypertension 3. Weight loss 4. Cataracts 14. When prescribing medroxyprogesterone (Depo Provera) injections, essential education would include advising of the following potential adverse drug effects: 1. Hypertension and dysuria 2. Depression and weight gain 3. Abdominal pain and constipation 4. Orthostatic hypotension and dermatitis 15. The medroxyprogesterone (Depo Provera) injection has a Black Box Warning due to: 1. The potential development of significant hypertension 2. Increased risk of strokes 3. Decreased bone density 4. The risk of a life-threatening rash such as Stevens-Johnson 16. Shana received her first medroxyprogesterone (Depo Provera) injection 6 weeks ago and calls the clinic with a concern that she has been having a light “period” off and on since receiving her Depo shot. What would be the management of Shana? 1. Reassurance that some spotting is normal the first few months of Depo and it should improve. 2. Schedule an appointment for an exam as this is not normal. 3. Prescribe 4 weeks of estrogen to treat the abnormal vaginal bleeding. 4. Order a pregnancy test and suggest she use a back-up method of contraception until she has her next shot. 17. William is a 62-year-old male who is requesting a prescription for sildenafil (Viagra). He should be screened for before prescribing sildenafil. 1. Renal dysfunction 2. Unstable coronary artery disease 3. Benign prostatic hypertrophy 4. History of priapism 18. Men who are prescribed sildenafil (Viagra) need ongoing monitoring for: 1. Development of chest pain or dizziness 2. Weight gain 3. Priapism 4. Renal function 19. Men who are prescribed an erectile dysfunction drug such as sildenafil (Viagra) should be warned about the risk for: 1. Impotence when combined with antihypertensives 2. Fatal hypotension if combined with nitrates 3. Weight gain if combined with antidepressants 4. All of the above 20. Androgens are indicated for: 1. Symptomatic treatment for male deficiency 2. Female libido, endometriosis, and postmenopausal symptoms 3. Increased muscle mass 4. Symptomatic treatment in both sexes for cancer and HIV 5. 1, 2, and 4 6. All of the above 21. Long-term use of androgens requires specific laboratory monitoring of: 1. Glucose, calcium, testosterone, and thyroid function 2. Calcium, testosterone, PSA, and liver function 3. Calcium, testosterone, PSA, liver function, glucose, and lipids 4. CBC, testosterone, PSA, and thyroid level 22. Effects of estrogen include: 1. Regulation of the menstrual cycle 2. Maintenance of bone density by increasing bone reabsorption 3. Maintenance of the normal structure of the skin and blood vessels 4. A and C 5. All of the above 23. Absolute contraindications that clinicians must consider when initiating estrogen therapy include: 1. Undiagnosed dysfunctional uterine bleeding 2. Deep vein or arterial thromboemboli within the prior year 3. Endometriosis 4. 1 and 2 5. All of the above 24. Patients taking hormonal contraceptives and hormone replacement therapy need to take the drug daily at the same time to prevent: 1. Nausea 2. Breakthrough bleeding 3. Breast tenderness 4. Pregnancy Chapter 23. Drugs Affecting the Integumentary System 1. Erik presents with one golden-crusted lesion at the site of an insect bite consistent with impetigo. His parents have limited finances and request the least expensive-treatment. Which medication would be the best choice for treatment? 1. Mupirocin (Bactroban) 2. Bacitracin and polymixin B (generic double antibiotic ointment) 3. Retapamulin (Altabax) 4. Oral cephalexin (Keflex) 2. Juakeem is a nasal methicillin resistant staphylococcus aureus (MRSA) carrier. Treatment to eradicate nasal MRSA is mupirocin. Patient education regarding treating nasal MRSA includes: 1. Take the oral medication exactly as prescribed. 2. Insert one-half of the dose in each nostril twice a day. 3. Alternate treating one nare in the morning and the other in the evening. 4. Nasal MRSA eradication requires at least 4 weeks of therapy, with up to 8 weeks needed in some patients. 3. Instructions for applying a topical antibiotic or antiviral ointment include: 1. Apply thickly to the infected area, spreading the medication well past the borders of the infection. 2. If the rash worsens, apply a thicker layer of medication to settle down the infection. 3. Wash hands before and after application of topical antimicrobials. 4. None of the above 4. When Sam used clotrimazole (Lotrimin AF) for athlete’s foot he developed a red, itchy rash consistent with a hypersensitivity reaction. He now has athlete’s foot again. What would be a good choice of antifungal for Sam? 1. Miconazole (Micatin) powder 2. Ketoconazole (Nizoral) cream 3. Terbinafine (Lamisil) cream 4. Griseofulvin (Grifulvin V) suspension 5. When prescribing griseofulvin (Grifulvin V) to treat tinea capitis it is critical to instruct the patient or parent to: 1. Mix the griseofulvin with ice cream before administering 2. Take the griseofulvin until the tinea clears, in approximately 4 to 5 weeks 3. Shampoo with baby shampoo daily while taking the griseofulvin 4. Griseofulvin is best absorbed if ingested with a high-fat food. 6. First-line therapy for treating topical fungal infections such as tinea corporis (ringworm) or tinea pedis (athlete’s foot) would be: 1. OTC topical azole (clotrimazole, miconazole) 2. Oral terbinafine 3. Oral griseofulvin microsize 4. Nystatin cream or ointment 7. When prescribing topical penciclovir (Denavir) for the treatment of herpes labialis (cold sores) patient education would include: 4. Use gloves to apply the malathion (Ovide). 20. When writing a prescription of permethrin 5% cream (Elimite) for scabies, patient education would include: 1. All members of the household and personal contacts should also be treated. 2. Infants should have permethrin applied from the neck down. 3. The permethrin is washed off after 10 to 20 minutes. 4. Permethrin is flammable and to avoid open flame while the medication is applied. Chapter 24. Drugs Used in Treating Infectious Diseases 1. Factors that place a patient at risk of developing an antimicrobial-resistant organism include: 1. Age over 50 years 2. School attendance 3. Travel within the U.S. 4. Inappropriate use of antimicrobials 2. Infants and young children are at higher risk of developing antibiotic-resistant infections due to: 1. Developmental differences in pharmacokinetics of the antibiotics in children 2. The fact that children this age are more likely to be in daycare and exposed to pathogens from other children 3. Parents of young children insisting on preventive antibiotics so they don’t miss work when their child is sick 4. Immunosuppression from the multiple vaccines they receive in the first 2 years of life 3. Providers should use an antibiogram when prescribing. An antibiogram is: 1. The other name for the Centers for Disease Control guidelines for prescribing antibiotics 2. An algorithm used for prescribing antibiotics for certain infections 3. The reference also known as the Pink Book, published by the Centers for Disease Control 4. A chart of the local resistance patterns to antibiotics developed by laboratories 4. There is often cross-sensitivity and cross-resistance between penicillins and cephalosporins because: 1. Renal excretion is similar in both classes of drugs. 2. When these drug classes are metabolized in the liver they both produce resistant enzymes. 3. Both drug classes contain a beta-lactam ring that is vulnerable to beta- lactamase-producing organisms. 4. There is not an issue with cross-resistance between the penicillins and cephalosporins. 5. Jonathan has been diagnosed with strep throat and needs a prescription for an antibiotic. He says the last time he had penicillin he developed a red, blotchy rash. An appropriate antibiotic to prescribe would be: 1. Penicillin VK, because his rash does not sound like a serious rash 2. Amoxicillin 3. Cefadroxil (Duricef) 4. Azithromycin 6. Sarah is a 25-year-old female who is 8 weeks pregnant and has a urinary tract infection. What would be the appropriate antibiotic to prescribe for her? 1. Ciprofloxacin (Cipro) 2. Amoxicillin (Trimox) 3. Doxycycline 4. Trimethoprim-sulfamethoxazole (Septra) 7. Pong-tai is a 12-month-old child who is being treated with amoxicillin for acute otitis media. His parents call the clinic and say he has developed diarrhea. The appropriate action would be to: 1. Advise the parents that some diarrhea is normal with amoxicillin and recommend probiotics daily. 2. Change the antibiotic to one that is less of a gastrointestinal irritant. 3. Order stool cultures for suspected viral pathogens not treated by the amoxicillin. 4. Recommend increased fluids and fiber in his diet. 8. Lauren is a 13-year-old child who comes to clinic with a 4-day history of cough, low-grade fever, and rhinorrhea. When she blows her nose or coughs the mucous is greenish-yellow. The appropriate antibiotic to prescribe would be: 1. Amoxicillin 2. Amoxicillin/clavulanate 3. TMP/SMZ (Septra) 4. None 9. Joanna had a small ventricle septal defect (VSD) repaired when she was 3 years old and has no residual cardiac problems. She is now 28 and is requesting prophylactic antibiotics for an upcoming dental visit. The appropriate antibiotic to prescribe according to current American College of Cardiology and American Heart Association guidelines is: 1. None, no antibiotic is required for dental procedures 2. Amoxicillin 2 grams 1 hour before the procedure 3. Ampicillin 2 grams IM or IV 30 minutes before the procedure 4. Azithromycin 1 gram 1 hour before the procedure 10. To prevent further development of antibacterial resistance it is recommended that fluoroquinolones be reserved for treatment of: 1. Urinary tract infections in young women 2. Upper respiratory infections in adults 3. Skin and soft tissue infections in adults 4. Community-acquired pneumonia in patients with comorbidities 11. Fluoroquinolones have a Black Box Warning regarding even months after treatment. 1. Renal dysfunction 2. Hepatic toxicity 3. Tendon rupture 4. Development of glaucoma 12. Janet was recently treated with clindamycin for an infection. She calls the advice nurse because she is having frequent diarrhea that she thinks may have blood in it. What would be the appropriate care for her? 1. Encourage increased fluids and fiber. 2. Assess her for pseudomembranous colitis. 3. Advise her to eat yogurt daily to help restore her gut bacteria. 4. Start her on an antidiarrheal medication. 13. Keng has chronic hepatitis that has led to mildly impaired liver function. He has an infection that would be best treated by a macrolide. Which would be the best choice for a patient with liver dysfunction? 1. Azithromycin (Zithromax) 2. Clarithromycin (Biaxin) 3. Erythromycin (E-mycin) 4. None of the above 14. Jamie has glucose-6-phosphate dehydrogenase deficiency (G6PD) and requires an antibiotic. Which class of antibiotics should be avoided in this patient? 1. Penicillins 2. Macrolides 3. Cephalosporins 4. Sulfonamides 15. If a patient is allergic to sulfonamide antibiotics, he or she will most likely have cross-sensitivity to: 1. Loop diuretics 2. Sulfonylureas 3. Thiazide diuretics 4. All of the above 16. Tetracyclines such as minocycline are safe to use in: 1. Pregnant women 2. Adolescents 3. Patients with renal dysfunction 4. Patients with hepatic dysfunction 17. Tetracyclines should not be prescribed to children younger than 8 years due to: 1. Risk of developing cartilage problems 2. Development of significant diarrhea 3. Risk of kernicterus 4. Adverse effects on bone growth 18. Nicole is a 16-year-old female who is taking minocycline for acne. She comes to the clinic complaining of a headache. What would be the plan of care? 1. Advise acetaminophen or ibuprofen as needed for headaches. 2. Prescribe sumatriptan (Imitrex) to be taken at the onset of the headache. 3. Evaluate her for pseudotremor cerebri. 4. Assess her caffeine intake and sleep patterns. corticosteroids. 2. Prednisone can be abruptly discontinued with no adverse effects. 3. Develop a tapering schedule to slowly wean Daniel off the prednisone. 4. Substitute the prednisone with another anti-inflammatory such as ibuprofen. 8. Patients with rheumatoid arthritis who are on chronic low-dose prednisone will need co- treatment with which medications to prevent further adverse effects? 1. A bisphosphonate 2. Calcium supplementation 3. Vitamin D 4. All of the above 9. Patients who are on or who will be starting chronic corticosteroid therapy need monitoring of: 1. Serum glucose 2. Stool culture 3. Folate levels 4. Vitamin B12 10. Patients who are on chronic long-term corticosteroid therapy need education regarding: 1. Receiving all vaccinations, especially the live flu vaccine 2. Reporting black tarry stools or abdominal pain 3. Eating a high carbohydrate diet with plenty of fluids 4. Small amounts of alcohol are generally tolerated. 11. All nonsteroidal anti-inflammatory drugs (NSAIDS) have an FDA Black Box Warning regarding: 1. Potential for causing life-threatening GI bleeds 2. Increased risk of developing systemic arthritis with prolonged use 3. Risk of life-threatening rashes, including Stevens-Johnson 4. Potential for transient changes in serum glucose 12. Jamie has fractured his ankle and has received a prescription for acetaminophen and hydrocodone (Vicodin). Education when prescribing Vicodin includes: 1. It is okay to double the dose of Vicodin if the pain is severe. 2. Vicodin is not habit-forming. 3. He should not take any other acetaminophen-containing medications. 4. Vicodin may cause diarrhea; increase his fluid intake. 13. When prescribing NSAIDS, a complete drug history should be conducted as NSAIDs interact with these drugs: 1. Omeprazole, a proton pump inhibitor 2. Combined oral contraceptives 3. Diphenhydramine, an antihistamine 4. Warfarin, an anticoagulant 14. Josefina is a 2-year-old child with acute otitis media and an upper respiratory infection. Along with an antibiotic she receives a recommendation to treat the ear pain with ibuprofen. What education would her parent need regarding ibuprofen? 1. They can cut an adult ibuprofen tablet in half to give Josefina. 2. The ibuprofen dose can be doubled for severe pain. 3. Josefina needs to be well-hydrated while taking ibuprofen. 4. Ibuprofen is completely safe in children with no known adverse effects. 15. Henry is 82 years old and takes two aspirin every morning to treat the arthritis pain in his back. He states the aspirin helps him to “get going” each day. Lately he has had some heartburn from the aspirin. After ruling out an acute GI bleed, what would be an appropriate course of treatment for Henry? 1. Add an H2 blocker such as ranitidine to his therapy. 2. Discontinue the aspirin and switch him to Vicodin for the pain. 3. Decrease the aspirin dose to one tablet daily. 4. Have Henry take an antacid 15 minutes before taking the aspirin each day. 16. The trial period to determine effective anti-inflammatory activity when starting a patient on aspirin for rheumatoid arthritis is: 1. 48 hours 2. 4 to 6 days 3. 4 weeks 4. 2 months 17. Patients prescribed aspirin therapy require education regarding the signs of aspirin toxicity. An early sign of aspirin toxicity is: 1. Black tarry stools 2. Vomiting 3. Tremors 4. Tinnitus 18. Monitoring a patient on a high-dose aspirin level includes: 1. Salicylate level 2. Complete blood count 3. Urine pH 4. All of the above 19. Patients who are on long-term aspirin therapy should have annually. 1. Complete blood count 2. Salicylate level 3. Amylase 4. Urine analysis Chapter 27. Anemia 1. Pernicious anemia is treated with: 1. Folic acid supplements 2. Thiamine supplements 3. Vitamin B12 4. Iron 2. Premature infants require iron supplementation with: 1. 10 mg/day of iron 2. 2 mg/kg per day until age 12 months 3. 7 mg/day in their diet 4. 1 mg/kg per day until they are receiving adequate intake of iron from foods 3. Breastfed infants should receive iron supplementation of: 1. 3 mg/kg per day 2. 6 mg/kg per day 3. 1 mg/kg per day 4. Breastfed babies do not need iron supplementation 4. Valerie presents to the clinic with menorrhagia. Her hemoglobin is 10.2 and her ferritin is 15 ng/mL. Initial treatment for her anemia would be: 1. 18 mg/day of iron supplementation 2. 6 mg/kg per day of iron supplementation 3. 325 mg ferrous sulfate per day 4. 325 mg ferrous sulfate tid 5. Chee is a 15-month-old male whose screening hemoglobin is 10.4 g/dL. Treatment for his anemia would be: 1. 18 mg/day of iron supplementation 2. 6 mg/kg per day of elemental iron 3. 325 mg ferrous sulfate per day 4. 325 mg ferrous sulfate tid 6. Monitoring for a patient taking iron to treat iron deficiency anemia is: 1. Hemoglobin, hematocrit, and ferritin 4 weeks after treatment is started 2. Complete blood count every 4 weeks throughout treatment 3. Annual complete blood count 4. Reticulocyte count in 4 weeks 7. Valerie has been prescribed iron to treat her anemia. Education of patients prescribed iron would include: 1. Take the iron with milk if it upsets her stomach. 2. Antacids may help with the nausea and GI upset caused by iron. 3. Increase fluids and fiber to treat constipation. 4. Iron is best tolerated if it is taken at the same time as her other medications. 8. Allie has just had her pregnancy confirmed and is asking about how to ensure a healthy baby. What is the folic acid requirement during pregnancy? 1. 40 mcg/day 2. 200 mcg/day 3. 800 mcg/day 4. 2 gm/day 9. Kyle has Crohn’s disease and has a documented folate deficiency. Drug therapy for folate deficiency anemia is: 1. Oral folic acid 1 to 2 mg per day 2. Oral folic acid 1 gram per day 3. IM folate weekly for at least 6 months 4. Oral folic acid 400 mcg daily 4. Neither 1 nor 2 9.Beta blockers are especially helpful for patients with exertional angina who also have: 1. Arrhythmias 2. Hypothyroidism 3. Hyperlipidemia 4. Atherosclerosis 10.Rapid-acting nitrates are important for all angina patients. Which of the following are true statements about their use? 1. These drugs are useful for immediate symptom relief when the patient is certain it is angina. 2. The dose is one sublingual tablet or spray every 5 minutes until the chest pain goes away. 3. Take one nitroglycerine tablet or spray at the first sign of angina; repeat every 5 minutes for no more than two doses. If chest pain is still not relieved, call 911. 4. All of the above 11.Isosorbide dinitrate is a long-acting nitrate given twice daily. The schedule for administration is 7 a.m. and 2 p.m. because: 1. Long-acting forms have a higher risk for toxicity. 2. Orthostatic hypotension is a common adverse effect. 3. It must be taken with milk or food. 4. Nitrate tolerance can develop. 12.Combinations of a long-acting nitrate and a beta blocker are especially effective in treating angina because: 1. Nitrates increase MOS and beta blockers increase MOD. 2. Their additive effects permit lower doses of both drugs and their adverse reactions cancel each other out. 3. They address the pathology of patients with exertional angina who have fixed atherosclerotic coronary heart disease. 4. All of the above 13.Drug choices to treat angina in older adults differ from those of younger adults only in: 1. Consideration of risk factors for diseases associated with and increased in aging 2. The placement of drug therapy as a treatment choice before lifestyle changes are tried 3. The need for at least three drugs in the treatment regimen because of the complexity of angina in the older adult 4. Those with higher risk for silent myocardial infarction 14.Which of the following drugs has been associated with increased risk for myocardial infarction in women? 1. Aspirin 2. Beta blockers 3. Estrogen replacement 4. Lipid-lowering agents 15.Cost of antianginal drug therapy should be considered in drug selection because of all of the following EXCEPT: 1. Patients often require multiple drugs 2. A large number of angina patients are older adults on fixed incomes 3. Generic formulations may be cheaper but are rarely bioequivalent 4. Lack of drug selectivity may result in increased adverse reactions 16.Situations that suggest referral to a specialist is appropriate include: 1. When chronic stable angina becomes unpredictable in its characteristics and precipitating factors 2. When a post-myocardial infarction patient develops new-onset angina 3. When standard therapy is not successful in improving exercise tolerance or reducing the incidence of angina 4. All of the above 17.The rationale for prescribing calcium blockers for angina can be based on the need for: 1. Increased inotropic effect in the heart 2. Increasing peripheral perfusion 3. Keeping heart rates high enough to ensure perfusion of coronary arteries 4. Help with rate control 18.Medications are typically started for angina patients when: 1. The first permanent EKG changes occur 2. The start of class I or II symptoms 3. The events trigger a trip to the emergency department 4. When troponin levels become altered 19.The most common cause of angina is: 1. Vasospasm of the coronary arteries 2. Atherosclerosis 3. Platelet aggregation 4. Low systemic oxygen 20.Ranolazine is used in angina patients to: 1. Dilate plaque-filled arteries 2. Inhibit platelet aggregation 3. Restrict late sodium flow in the myocytes 4. Induce vasoconstriction in the periphery to open coronary vessels 21.When is aspirin (ASA) used in angina patients? 1. All angina patients should be taking ASA unless it is contraindicated for allergy or other medical reasons. Chapter 29. Anxiety and Depression 1. Common mistakes practitioners make in treating anxiety disorders include: 1. Switching medications after an 8- to 12-week trial 2. Maximizing dosing of antianxiety medications 3. Encouraging exercise and relaxation therapy before starting medication 4. Thinking a partial response to medication is acceptable 2. An appropriate first-line drug to try for mild to moderate generalized anxiety disorder would be: 1. Alprazolam (Xanax) 2. Diazepam (Valium) 3. Buspirone (Buspar) 4. Amitriptyline (Elavil) 3. An appropriate drug to initially treat panic disorder is: 1. Alprazolam (Xanax) 2. Diazepam (Valium) 3. Buspirone (Buspar) 4. Amitriptyline (Elavil) 4. Prior to starting antidepressants, patients should have laboratory testing to rule out: 1. Hypothyroidism 2. Anemia 3. Diabetes mellitus 4. Low estrogen levels 5. David is a 34-year-old patient who is starting on paroxetine (Paxil) for depression. David’s education regarding his medication would include: 1. Paroxetine may cause intermittent diarrhea. 2. He may experience sexual dysfunction beginning a month after he starts therapy. 3. He may have constipation and he should increase fluids and fiber. 4. Paroxetine has a long half-life so he may occasionally skip a dose. 6. Jamison has been prescribed citalopram (Celexa) to treat his depression. Education regarding how quickly selective serotonin reuptake inhibitor (SSRI) antidepressants work would be: 1. Appetite and concentration improve in the first 1 to 2 weeks. 2. Sleep should improve almost immediately upon starting citalopram. 3. Full response to the SSRI may take 2 to 4 months after he reaches the full therapeutic dose. 4. His dysphoric mood will improve in 1 to 2 weeks. 7. An appropriate drug for the treatment of depression with anxiety would be: 1. Alprazolam (Xanax) 2. Escitalopram (Lexapro) 3. Buspirone (Buspar) 4. Amitriptyline (Elavil) 8. An appropriate first-line drug for the treatment of depression with fatigue and low energy would be: 1. Venlafaxine (Effexor) 2. Escitalopram (Lexapro) 3. Buspirone (Buspar) 2. Theophylline 3. Low-dose inhaled corticosteroids 4. Long-acting beta-2-agonists 7. Monitoring a patient with persistent asthma includes: 1. Monitoring how frequently the patient has an upper respiratory infection (URI) during treatment 2. Monthly in-office spirometry testing 3. Determining if the patient has increased use of his or her long-acting beta-2- agonist due to exacerbations 4. Evaluating the patient every 1 to 6 months to determine if the patient needs to step up or down in their therapy 8. Asthma exacerbations at home are managed by the patient by: 1. Increasing frequency of beta-2-agonists and contacting their provider 2. Doubling inhaled corticosteroid doses 3. Increasing frequency of beta-2-agonists 4. Starting montelukast (Singulair) 9. Patients who are at risk of a fatal asthma attack include patients: 1. With moderate persistent asthma 2. With a history of requiring intubation or ICU admission for asthma 3. Who are on daily inhaled corticosteroid therapy 4. Who are pregnant 10. Pregnant patients with asthma may safely use throughout their pregnancy. 1. Oral terbutaline 2. Prednisone 3. Inhaled corticosteroids (budesonide) 4. Montelukast (Singulair) 11. One goal of asthma management in children is: 1. They independently manage their asthma 2. Participation in school and sports activities 3. No exacerbations 4. Minimal use of inhaled corticosteroids 12. Medications used in the management of patients with chronic obstructive pulmonary disease (COPD) include: 1. Inhaled beta-2-agonists 2. Inhaled anticholinergics (ipratropium) 3. Inhaled corticosteroids 4. All of the above 13. Patients with a COPD exacerbation may require: 1. Doubling of inhaled corticosteroid dose 2. Systemic corticosteroid burst 3. Continuous inhaled beta-2-agonists 4. Leukotriene therapy 14. Patients with COPD require monitoring of: 1. Beta-2-agonist use 2. Serum electrolytes 3. Blood pressure 4. Neuropsychiatric effects of montelukast 15. Education of patients with COPD who use inhaled corticosteroids includes: 1. Doubling the dose at the first sign of a URI 2. Using their inhaled corticosteroid first and then their bronchodilator 3. Rinsing their mouth after use 84 Abstaining from smoking for at least 30 minutes after using . 16. Education for patients who use an inhaled beta-agonist and an inhaled corticosteroid includes: 1. Use the inhaled corticosteroid first, followed by the inhaled beta-agonists. 2. Use the inhaled beta-agonist first, followed by the inhaled corticosteroid. 3. Increase fluid intake to 3 liters per day. 4. Avoid use of aspirin or ibuprofen while using inhaled medications. Chapter 31. Contraception 1. Women who are taking an oral contraceptive containing the progesterone drospirenone may require monitoring of: 1. Hemoglobin 2. Serum calcium 3. White blood count 4. Serum potassium 2. The mechanism of action of oral combined contraceptives that prevents pregnancy is: 1. Estrogen prevents the luteinizing hormone surge necessary for ovulation. 2. Progestins thicken cervical mucus and slow tubal motility. 3. Estrogen thins the endometrium making implantation difficult. 4. Progestin suppresses follicle stimulating hormone release. 3. To improve actual effectiveness of oral contraceptives women should be educated regarding: 1. Use of a back-up method if they have vomiting or diarrhea during a pill packet 2. Doubling pills if they have diarrhea during the middle of a pill pack 3. The fact that they will have a normal menstrual cycle if they miss two pills 4. The fact that mid-cycle spotting is not normal and the provider should be contacted immediately 4. A contraindication to the use of combined contraceptives is: 1. Adolescence (not approved for this age) 2. A history of clotting disorder 3. Recent pregnancy 4. Being overweight 5. Obese women may have increased risk of failure with which contraceptive method? 1. Combined oral contraceptives 2. Progestin-only oral contraceptive pill 3. Injectable progestin 4. Combined topical patch 6. Ashley comes to the clinic with a request for oral contraceptives. She has successfully used oral contraceptives before and has recently started dating a new boyfriend so would like to restart contraception. She denies recent intercourse and has a negative urine pregnancy test in the clinic. An appropriate plan of care would be: 1. Recommend she return to the clinic at the start of her next menses to get a Depo Provera shot. 2. Prescribe oral combined contraceptives and recommend she start them at the beginning of her next period and use a back-up method for the first 7 days. 3. Prescribe oral contraceptives and have her start them the same day as the visit with a back-up method used for the first 7 days. 4. Discuss the advantages of using the topical birth control patch and recommend she consider using the patch. 7. When discussing with a patient the different start methods used for oral combined contraceptives, the advantage of a Sunday start over the other start methods is: 1. Immediate protection against pregnancy the first week of using the pill 2. No back-up method is needed when starting 3. Menses occur during the week 4. They can start the pill on the Sunday after the office visit 8. The topical patch combined contraceptive (Ortho Evra) is: 1. Started on the first day of the menstrual cycle 2. Recommended for women over 200 pounds 3. Not as effective as oral combined contraceptives 4. Known to have more adverse effects, such as nausea, than the oral combined contraceptives 9. Progesterone-only pills are recommended for women who: 1. Are breastfeeding 2. Have a history of migraine 3. Have a medical history that contradicts the use of estrogen 4. All of the above 10. Women who are prescribed progestin-only contraception need education regarding which common adverse drug effects? 1. Increased migraine headaches 2. Increased risk of developing blood clots 3. Irregular vaginal bleeding for the first few months 4. Increased risk for hypercalcemia 11. An advantage of using the NuvaRing vaginal ring for contraception is: 1. It does not require fitting and is easy to insert. 2. It is inserted once a week, eliminating the need to remember to take a daily pill. 14. Jim presents with fungal infection of two of his toenails (onychomycosis). Treatment for fungal infections of the nail includes: 1. Miconazole cream 2. Ketoconazole cream 3. Oral griseofulvin 4. Mupirocin cream 15. Scabies treatment for a 4-year-old child includes a prescription for: 1. Permethrin 5% cream applied from the neck down 2. Pyrethrin lotion 3. Lindane 1% shampoo 4. All of the above 16. Vanessa has been diagnosed with scabies. Her education would include: 1. She should apply the scabies treatment cream for an hour and wash it off. 2. Scabies may need to be retreated in a week after initial treatment. 3. All members of the household and close personal contacts should be treated. 4. Malathion is flammable and she should take care until the solution dries. 17. Catherine has head lice and her mother is asking about what products are available that are not neurotoxic. The only non-neurotoxin head lice treatment is: 1. Permethrin 1% (Nix) 2. Lindane shampoo 3. Malathion (Ovide) 4. Benzoyl alcohol (Ulesfia) 18. Rick has male pattern baldness on the vertex of his head and has been using Rogaine for 2 months. He asks how effective minoxidil (Rogaine) is. Minoxidil: 1. Provides a permanent solution to male pattern baldness if used for at least 4 months 2. Will show results after 4 months of twice-a-day use 3. May not work for Rick’s type of baldness 4. Works better if he also uses hydrocortisone cream daily on his scalp 33. Diabetes Mellitus 1. Type 1 diabetes results from autoimmune destruction of the beta cells. Eighty-five to 90% of type 1 diabetics have: 1. Autoantibodies to two tyrosine phosphatases 2. Mutation of the hepatic transcription factor on chromosome 12 3. A defective glucokinase molecule due to a defective gene on chromosome 7p 4. Mutation of the insulin promoter factor 2. Type 2 diabetes is a complex disorder involving: 1. Absence of insulin production by the beta cells 2. A suboptimal response of insulin-sensitive tissues in the liver 3. Increased levels of glucagon-like peptide in the postprandial period 4. Too much fat uptake in the intestine 3. Diagnostic criteria for diabetes include: 1. Fasting blood glucose greater than 140 mg/dl on two occasions 2. Postprandial blood glucose greater than 140 mg/dl 3. Fasting blood glucose 100 to 125 mg/dl on two occasions 4. Symptoms of diabetes plus a casual blood glucose greater than 200 mg/dl 4. Routine screening of asymptomatic adults for diabetes is appropriate for: 1. Individuals who are older than 45 and have a BMI of less than 25 kg/m2 2. Native Americans, African Americans, and Hispanics 3. Persons with HDL cholesterol greater than 100 mg/dl 4. Persons with prediabetes confirmed on at least two occasions 5. Screening for children who meet the following criteria should begin at age 10 and occur every 3 years thereafter: 1. BMI above the 85th percentile for age and sex 2. Family history of diabetes in first- or second-degree relative 3. Hypertension based on criteria for children 4. Any of the above 6. Insulin is used to treat both types of diabetes. It acts by: 1. Increasing beta cell response to low blood-glucose levels 2. Stimulating hepatic glucose production 3. Increasing peripheral glucose uptake by skeletal muscle and fat 4. Improving the circulation of free fatty acids 7. Adam has type 1 diabetes and plays tennis for his university. He exhibits a knowledge deficit about his insulin and his diagnosis. He should be taught that: 1. He should increase his carbohydrate intake during times of exercise. 2. Each brand of insulin is equal in bioavailability, so buy the least expensive. 3. Alcohol produces hypoglycemia and can help control his diabetes when taken in small amounts. 4. If he does not want to learn to give himself injections, he may substitute an oral hypoglycemic to control his diabetes. 8. Insulin preparations are divided into categories based on onset, duration, and intensity of action following subcutaneous injection. Which of the following insulin preparations has the shortest onset and duration of action? 1. Lispro 2. Glulisine 3. Glargine 4. Detemir 9. The drug of choice for type 2 diabetics is metformin. Metformin: 1. Decreases glycogenolysis by the liver 2. Increases the release of insulin from beta cells 3. Increases intestinal uptake of glucose 4. Prevents weight gain associated with hyperglycemia 10. Before prescribing metformin, the provider should: 1. Draw a serum creatinine level to assess renal function. 2. Try the patient on insulin. 3. Prescribe a thyroid preparation if the patient needs to lose weight. 4. All of the above 11. Sulfonylureas may be added to a treatment regimen for type 2 diabetics when lifestyle modifications and metformin are insufficient to achieve target glucose levels. Sulfonylureas have been moved to Step 2 therapy because they: 1. Increase endogenous insulin secretion 2. Have a significant risk for hypoglycemia 3. Address the insulin resistance found in type 2 diabetics 4. Improve insulin binding to receptors 12. Dipeptidyl peptidase-4 inhibitors (gliptins) act on the incretin system to improve glycemic control. Advantages of these drugs include: 1. Better reduction in glucose levels than other classes 2. Less weight gain than sulfonylureas 3. Low risk for hypoglycemia 4. Can be given twice daily 13. Control targets for patients with diabetes include: 1. HbA1C between 7 and 8 2. Fasting blood glucose levels between 100 and 120 mg/dl 3. Blood pressure less than 130/80 mm Hg 4. LDL lipids less than 130 mg/dl 14. Establishing glycemic targets is the first step in treatment of both types of diabetes. For type 1 diabetes: 1. Tight control/intensive therapy can be given to adults who are willing to test their blood glucose at least twice daily. 2. Tight control is acceptable for older adults if they are without complications. 3. Plasma glucose levels are the same for children as adults. 4. Conventional therapy has a fasting plasma glucose target between 120 and 150 mg/dl. 15. Treatment with insulin for type 1 diabetics: 1. Starts with a total daily dose of 0.2 to 0.4 units per kg of body weight 2. Divides the total doses into three injections based on meal size 3. Uses a total daily dose of insulin glargine given once daily with no other insulin required 4. Is based on the level of blood glucose 16. When the total daily insulin dose is split and given twice daily, which of the following rules may be followed? 1. Give two-thirds of the total dose in the morning and one-third in the evening. 2. Give 0.3 units per kg of premixed 70/30 insulin with one-third in the morning and two-thirds in the evening. 1. Fasting blood glucose 2. HbA1C 3. Thyroid function tests 4. Electrocardiograms 31. Allison is an 18-year-old college student with type 1 diabetes. She is on NPH twice daily and Novolog before meals. She usually walks for 40 minutes each evening as part of her exercise regimen. She is beginning a 30-minute swimming class three times a week at 1 p.m. What is important for her to do with this change in routine? 1. Delay eating the midday meal until after the swimming class. 2. Increase the morning dose of NPH insulin on days of the swimming class. 3. Adjust the morning insulin injection so that the peak occurs while swimming. 4. Check glucose level before, during, and after swimming. 32. Allison is an 18-year-old college student with type 1 diabetes. Allison’s pre-meal BG at 11:30 a.m. is 130. She eats an apple and has a sugar-free soft drink. At 1 p.m. before swimming her BG is 80. What should she do? 1. Proceed with the swimming class. 2. Recheck her BG immediately. 3. Eat a granola bar or other snack with CHO. 4. Take an additional dose of insulin. 33. Bart is a patient is a 67-year-old male with T2 DM. He is on glipizide and metformin. He presents to the clinic with confusion, sluggishness, and extreme thirst. His wife tells you Bart does not follow his meal plan or exercise regularly, and hasn’t checked his BG for 1 week. A random glucose is drawn and it is 500. What is a likely diagnosis based on preliminary assessment? 1. Diabetic keto acidosis (DKA) 2. Hyperglycemic hyperosmolar syndrome (HHS) 3. Infection 4. Hypoglycemia 34. What would one expected assessment finding be for hyperglycemic hyperosmolar syndrome? 1. Low hemoglobin 2. Ketones in the urine 3. Deep, labored breathing 4. pH of 7.35 35. A patient on metformin and glipizide arrives at her 11:30 a.m. clinic appointment diaphoretic and dizzy. She reports taking her medication this morning and ate a bagel and coffee for breakfast. BP is 110/70 and random finger-stick glucose is 64. How should this patient be treated? 1. 12 oz apple juice with 1 tsp sugar 2. 10 oz diet soda 3. 8 oz milk or 4 oz orange juice 4. 4 cookies and 8 oz chocolate milk Chapter 34. Gastroesophageal Reflux and Peptic Ulcer Disease 1. Gastroesophageal reflux disease may be aggravated by the following medication that affects lower esophageal sphincter (LES) tone: 1. Calcium carbonate 2. Estrogen 3. Furosemide 4. Metoclopramide 2. Lifestyle changes are the first step in treatment of gastroesophageal reflux disease (GERD). Food or drink that may aggravate GERD include: 1. Eggs 2. Caffeine 3. Chocolate 4. Soda pop 3. Metoclopramide improves gastroesophageal reflux disease symptoms by: 1. Reducing acid secretion 2. Increasing gastric pH 3. Increasing lower esophageal tone 4. Decreasing lower esophageal tone 4. Antacids treat gastroesophageal reflux disease by: 1. Increasing lower esophageal tone 2. Increasing gastric pH 3. Inhibiting gastric acid secretion 4. Increasing serum calcium level 5. When treating patients using the “Step-Down” approach the patient with gastroesophageal reflux disease is started on first. 1. Antacids 2. Histamine2 receptor antagonists 3. Prokinetics 4. Proton pump inhibitors 6. If a patient with symptoms of gastroesophageal reflux disease states that he has been self-treating at home with OTC ranitidine daily, the appropriate treatment would be: 1. Prokinetic (metoclopramide) for 4 to 8 weeks 2. Proton pump inhibitor (omeprazole) for 12 weeks 3. Histamine2 receptor antagonist (ranitidine) for 4 to 8 weeks 4. Cytoprotective drug (misoprostol) for 2 weeks 7. If a patient with gastroesophageal reflux disease who is taking a proton pump inhibitor daily is not improving, the plan of care would be: 1. Prokinetic (metoclopramide) for 8 to 12 weeks 2. Proton pump inhibitor (omeprazole) twice a day for 4 to 8 weeks 3. Histamine2 receptor antagonist (ranitidine) for 4 to 8 weeks 4. Cytoprotective drug (misoprostol) for 4 to 8 weeks 8. The next step in treatment when a patient has been on proton pump inhibitors twice daily for 12 weeks and not improving is: 1. Add a prokinetic (metoclopramide) 2. Referral for endoscopy 3. Switch to another proton pump inhibitor 4. Add a cytoprotective drug 9. Infants with reflux are initially treated with: 1. Histamine2 receptor antagonist (ranitidine) 2. Proton pump inhibitor (omeprazole) 3. Anti-reflux maneuvers (elevate head of bed) 4. Prokinetic (metoclopramide) 10. Long-term use of proton pump inhibitors may lead to: 1. Hip fractures in at-risk persons 2. Vitamin B6 deficiency 3. Liver cancer 4. All of the above 11. An acceptable first-line treatment for peptic ulcer disease with positive H. pylori test is: 1. Histamine2 receptor antagonists for 4 to 8 weeks 2. Proton pump inhibitor bid for 12 weeks until healing is complete 3. Proton pump inhibitor bid plus clarithromycin plus amoxicillin for 14 days 4. Proton pump inhibitor bid and levofloxacin for 14 days 12. Treatment failure in patients with peptic ulcer disease associated with H. pylori may be because of: 1. Antimicrobial resistance 2. An ineffective antacid 3. Overuse of proton pump inhibitors 4. All of the above 13. If a patient with H. pylori-positive peptic ulcer disease fails first-line therapy, the second-line treatment is: 1. Proton pump inhibitor bid plus metronidazole plus tetracycline plus bismuth subsalicylate for 14 days 2. Test H. pylori for resistance to common treatment regimens 3. Proton pump inhibitor plus clarithromycin plus amoxicillin for 14 days 4. Proton pump inhibitor and levofloxacin for 14 days 14. After H. pylori treatment is completed, the next step in peptic ulcer disease therapy is: 1. Testing for H. pylori eradication with a serum ELISA test 2. Endoscopy by a specialist 3. A proton pump inhibitor for 8 to 12 weeks until healing is complete 4. All of the above Chapter 39. Hyperlipidemia 1. The overall goal of treating hyperlipidemia is: 1. Maintain an LDL level of less than 160 mg/dL 15. When are statins traditionally ordered to be taken? 1. At bedtime 2. At noon 3. At breakfast 4. With the evening meal 16. Which the following persons should not have a statin medication ordered? 1. Someone with 3 first- or second-degree family members with history of muscle issues when started on statins 2. Someone with high lipids, but low BMI 3. Premenopausal woman with recent history of hysterectomy 4. Prediabetic male with known metabolic syndrome 17. Fiber supplements are great options for elderly patients who have the concurrent problem of: 1. End-stage renal failure on fluid restriction 2. Recurrent episodes of diarrhea several times a day 3. Long-term issues of constipation 4. Needing to take multiple medications around the clock every 2 hours 18. What is considered the order of statin strength from lowest effect to highest? 1. Lovastatin, Simvastatin, Rosuvastatin 2. Rosuvastatin, Lovastatin, Atorvastatin 3. Atorvastatin, Rosuvastatin, Simvastatin 4. Simvastatin, Atorvastatin, Lovastatin Chapter 43. Smoking Cessation 1. Nicotine withdrawal symptoms include: 1. Nervousness 2. Increased appetite 3. Difficulty concentrating 4. All of the above 2. If a patient wants to quit smoking, nicotine replacement therapy is recommended if the patient: 1. Smokes more than 10 cigarettes a day 2. Smokes within 30 minutes of awakening in the morning 3. Smokes when drinking alcohol 4. All of the above 3. Instructions for a patient who is starting nicotine replacement therapy include: 1. Smoke less than 10 cigarettes a day when starting nicotine replacement. 2. Nicotine replacement will help with the withdrawal cravings associated with quitting tobacco. 3. Nicotine replacement can be used indefinitely. 4. Nicotine replacement therapy is generally safe for all patients. 4. Nicotine replacement therapy should not be used in which patients? 1. Pregnant women 2. Patients with worsening angina pectoris 3. Patients who have just suffered an acute myocardial infarction 4. All of the above 5. Instructions for the use of nicotine gum include: 1. Chew the gum quickly to get a peak effect. 2. The gum should be “parked” in the buccal space between chewing. 3. Acidic drinks such as coffee help with the absorption of the nicotine. 4. The highest abstinence rates occur if the patient chews the gum when he or she is having cravings. 6. Patients who choose the nicotine lozenge to assist in quitting tobacco should be instructed: 1. Chew the lozenge well. 2. Drink at least 8 ounces of water after the lozenge dissolves. 3. Use one lozenge every 1 to 2 hours (at least nine per day with a maximum of 20 per day). 4. A tingling sensation in the mouth should be reported to the provider. 7. Transdermal nicotine replacement (the patch) is an effective choice in tobacco cessation because: 1. The patch provides a steady level of nicotine without reinforcing oral aspects of smoking. 2. There is the ability to “fine tune” the amount of nicotine that is delivered to the patient at any one time. 3. There is less of a problem with nicotine toxicity than other forms of nicotine replacement. 4. Transdermal nicotine is safer in pregnancy. 8. The most common adverse effect of the transdermal nicotine replacement patch is: 1. Nicotine toxicity 2. Tingling at the site of patch application 3. Skin irritation under the patch site 4. Life-threatening dysrhythmias 9. If a patient is exhibiting signs of nicotine toxicity when using transdermal nicotine, they should remove the patch and: 1. Wash the area thoroughly with soap and water. 2. Flush the area with clear water. 3. Reapply a new patch in 8 hours. 4. Take acetaminophen for the headache associated with toxicity. 10. When a patient is prescribed nicotine nasal spray for tobacco cessation, instructions include: 1. Inhale deeply with each dose to ensure deposition in the lungs. 2. The dose is one to two sprays in each nostril per hour, not to exceed 40 sprays per day. 3. If they have a sensation of “head rush” this indicates the medication is working well. 4. Nicotine spray may be used for up to 12 continuous months. 11. If prescribing bupropion (Zyban) for tobacco cessation, the instructions to the patient include: 1. Bupropion (Zyban) is started 1 to 2 weeks before the quit date. 2. Nicotine replacement products should not be used with bupropion. 3. If they smoke when taking bupropion they may have increased anxiety and insomnia. 4. Because they are not using bupropion as an antidepressant, they do not need to worry about increased suicide ideation when starting therapy. 12. Varenicline (Chantix) may be prescribed for tobacco cessation. Instructions to the patient who is starting varenicline include: 1. The maximum time varenicline can be used is 12 weeks. 2. Nausea is a sign of varenicline toxicity and should be reported to the provider. 3. The starting regimen for varenicline is start taking 1 mg twice a day a week before the quit date. 4. Neuropsychiatric symptoms may occur. 13. The most appropriate smoking cessation prescription for pregnant women is: 1. A nicotine replacement patch at the lowest dose available 2. Bupropion (Zyban) 3. Varenicline (Chantix) 4. Nonpharmacologic measures Chapter 49. Men as Patients 1. The factor that has the greatest effect on males developing male sexual characteristics is: 1. Cultural beliefs 2. Effective male role models 3. Adequate intake of testosterone in the diet 4. Androgen production 2. When assessing a male for hypogonadism prior to prescribing testosterone replacement, serum testosterone levels are drawn: 1. Without regard to time of day 2. First thing in the morning 3. Late afternoon 4. In the evening 3. Some research supports that testosterone replacement therapy may be indicated in which of the following diagnoses in men? 1. Age-related decrease in cognitive functioning 2. Metabolic syndrome 3. Decreased muscle mass in aging men 4. All of the above 4. The goal of testosterone replacement therapy is: 1. Absence of all hypogonadism symptoms 2. Testosterone levels in the mid-normal range 1 week after an injection 3. Testosterone levels in the mid-normal range just prior to the next injection 4. Avoidance of high serum testosterone levels during therapy 2. Mixing the medication with a couple of ounces of formula and putting it in a bottle 3. Discontinuing the antibiotic if diarrhea occurs 4. Calling for an antibiotic change if the infant chokes and sputters during administration 9. To increase adherence in pediatric patients a prescription medication should: 1. Have a short half-life 2. Be the best tasting of the effective drugs 3. Be the least concentrated form of the medication 4. Be administered 3 or 4 times a day 10. Janie is a 5-month-old breastfed infant with a fever. Treatment for her fever may include: 1. “Baby” aspirin 2. Acetaminophen suppository 3. Ibuprofen suppository 4. Alternating acetaminophen and ibuprofen Chapter 51. Geriatric Patients 1. Principles of prescribing for older adults include: 1. Avoiding prescribing any newer high-cost medications 2. Starting at a low dose and increasing the dose slowly 3. Keeping the total dose at a lower therapeutic range 4. All of the above 2. Sadie is a 90-year-old patient who requires a new prescription. What changes in drug distribution with aging would influence prescribing for Sadie? 1. Increased volume of distribution 2. Decreased lipid solubility 3. Decreased plasma proteins 4. Increased muscle-to-fat ratio 3. Glen is an 82-year-old patient who needs to be prescribed a new drug. What changes in elimination should be taken into consideration when prescribing for Glen? 1. Increased glomerular filtration rate (GFR) will require higher doses of some renally excreted drugs. 2. Decreased tubular secretion of medication will require dosage adjustments. 3. Thin skin will cause increased elimination via sweat. 4. Decreased lung capacity will lead to measurable decreases in lung excretion of drugs. 4. A medication review of an elderly person’s medications involves: 1. Asking the patient to bring a list of current prescription medications to the visit 2. Having the patient bring all of their prescription, over-the-counter, and herbal medications to the visit 3. Asking what other providers are writing prescriptions for them 4. All of the above 5. Steps to avoid polypharmacy include: 1. Prescribing two or fewer drugs from each drug class 2. Reviewing a complete drug history every 12 to 18 months 3. Encouraging the elderly patient to coordinate their care with all of their providers 4. Evaluating for duplications in drug therapy and discontinuing any duplications 6. Robert is a 72-year-old patient who has hypertension and angina. He is at risk for common medication practices seen in the elderly including: 1. Use of another person’s medications 2. Hoarding medications 3. Changing his medication regimen without telling his provider 4. All of the above 7. To improve positive outcomes when prescribing for the elderly the nurse practitioner should: 1. Assess cognitive functioning in the elder 2. Encourage the patient to take a weekly “drug holiday” to keep drug costs down 3. Encourage the patient to cut drugs in half with a knife to lower costs 4. All of the above 8. When an elderly diabetic patient is constipated the best treatment options include: 1. Mineral oil 2. Bulk-forming laxatives such as psyllium 3. Stimulant laxatives such as senna 4. Stool softeners such as docusate 9. Delta is an 88-year-old patient who has mild low-back pain. What guidelines should be followed when prescribing pain management for Delta? 1. Keep the dose of oxycodone low to prevent development of tolerance. 2. Acetaminophen is the first-line drug of choice. 3. Avoid prescribing NSAIDs. 4. Add in a short-acting benzodiazepine for a synergistic effect on pain. 10. Robert is complaining of poor sleep. Medications that may contribute to sleep problems in the elderly include: 1. Diuretics 2. Trazodone 3. Clonazepam 4. Levodopa 11. The GFRs for a 91-year-old woman who weighs 93 pounds and is 5'1" with a serum creatinine of 1.1, and for a 202-pound, 25-year-old male who is 5'9" with the same serum creatinine according to the Cockcroft Gault formula are: 1. 25ml/ min and 133 mL/min respectively 2. 25 mL/min and 103 mL/min respectively 3. 22 ml/min and 133 mL/min respectively 4. 22 ml/min and 103 mL/min respectively 12. In geriatric patients, the percentage of body fat is increased. What are the pharmacologic implications of this physiologic change? 1. A lipid-soluble medication will be eliminated more quickly and not work as well. 2. A lipid-soluble medication will accumulate in fat tissue and its duration of action may be prolonged. 3. Absorption of lipid-soluble drugs is impaired in older adults. 4. The bioavailability of the lipid-soluble drug will be increased in older adults. 13. All of the following statements about the Beer’s List are true except: 1. It is a list of medications or medication classes that should generally be avoided in persons 65 years or older because they are either ineffective or they pose unnecessarily high risk for older persons and a safer alternative is available. 2. It is derived from the expert opinion of one geriatrician and is not evidence-based. 3. These criteria have been adopted by the Centers for Medicare and Medicaid Services for regulation of long-term care facilities. 4. These criteria are directed at the general population of patients over 65 years of age and do not take disease states into consideration. 14. You are reviewing the data from several meta-analyses that addressed the most common causes of adverse drug reactions in the older adult. Which of the following would you find to be decreased and the most common cause of these problems in older adults? 1. Body fat content 2. Liver function 3. Renal function/clearance 4. Plasma albumin levels 15. Which of the following is not consistent with the rules for geriatric prescribing: 1. Half-life will be longer in older adults 2. Steady state is reached more quickly in the older adult 3. Reduce the number of drugs in the patient's regimen whenever possible 4. Adverse drug responses present atypically in the older adult
Docsity logo



Copyright © 2024 Ladybird Srl - Via Leonardo da Vinci 16, 10126, Torino, Italy - VAT 10816460017 - All rights reserved